Preview (15 of 99 pages)

NCLEX-RN
Part 3
401. In an interview for suspected child abuse, the child's mother openly discusses her feelings.
She feels her husband is too aggressive in disciplining their child. The child's father states, "Being a school
custodian, I see kids every day that are bad because they did not get enough discipline at home. That will
not happen to our child." Based on this remark, the nurse would make the following nursing diagnosis:
A. Fear related to retaliation by the father
B. Actual injury related to poor impulse control by the father
C. Ineffective coping
D. Altered family process related to physical abuse
Answer: D
Explanation:
(A) There is no evidence of fear as the child is unable to communicate.
(B) There is actual injury, but the parents have not yet admitted causing the child's injuries.
(C) This diagnosis is incomplete. There is no specific ineffective coping behavior identified in this nursing
diagnosis.
(D) Altered family process best describes the family dynamics in this situation. The parents have admitted
severe disciplinary action.
402. A female client has been recently diagnosed as bipolar. She has taken lithium for the past several
weeks to control mania. What must be included in client education regarding lithium toxicity?
A. Maintain a normal diet; however, limit salt intake to no more than 3 g/day.
B. Take lithium between meals to increase absorption.
C. Withhold lithium if experiencing diarrhoea, vomiting, or diaphoresis.
D. For pain or fever, avoid aspirin or acetaminophen (Tylenol). Nonsteroidal anti inflammatory drugs are
preferred.
Answer: C
Explanation:
(A) The client should maintain a normal diet including normal salt intake. A low-sodium diet can cause
lithium retention, leading to toxicity.
(B) Lithium must be taken with meals because it is irritating to the gastric mucosa.

(C) Diarrhoea, vomiting, or diaphoresis can cause dehydration, which will increase lithium blood levels. If
these symptoms occur, the nurse should instruct the client to withhold lithium.
(D) Lithium is not to be taken with over-the-counter drugs without specific instruction. Some drugs raise
lithium levels, whereas others lower lithium levels.
403. A 56-year-old psychiatric inpatient has had recurring episodes of depression and chronic low selfesteem. She feels that her family does not want her around, experiences a sense of helplessness, and has a
negative view of herself. To assist the client in focusing on her strengths and positive traits, a strategy used
by the nurse would be to:
A. Tell the client to attend all structured activities on the unit
B. Encourage or direct client to attend activities that offer simple methods to attain success
C. Increase the client's self-esteem by asking that she make all decisions regarding attendance in group
activities
D. Not allow any dependent behaviors by the client because she must learn independence and will have to
ask for any assistance from staff
Answer: B
Explanation:
(A) The nurse should encourage activities gradually, as client's energy level and tolerance for shared
activities improve.
(B) Activities that focus on strengths and accomplishments, with uncomplicated tasks, minimize failure
and increase self-worth.
(C) Asking a client to set a goal to make all decisions about attending group activities is unrealistic, and
such decisions are not always under the client's control; this sets up the client for further failure and
possibly decreased self-worth.
(D) Encouragement toward independence does promote increased feelings of selfworth; however, clients
may need assistance with decision making and problem solving for various situations and on an individual
basis.
404. The physician recommends immediate hospital admission for a client with PIH. She says to the nurse,
"It's not so easy for me to just go right to the hospital like that." After acknowledging her feelings, which
of these approaches by the nurse would probably be best?
A. Stress to the client that her husband would want her to do what is best for her health.
B. Explore with the client her perceptions of why she is unable to go to the hospital.

C. Repeat the physician's reasons for advising immediate hospitalization.
D. Explain to the client that she is ultimately responsible for her own welfare and that of her baby.
Answer: B
Explanation:
(A) This answer does not hold the client accountable for her own health.
(B) The nurse should explore potential reasons for the client's anxiety: are there small children at home, is
the husband out of town? The nurse should aid the client in seeking support or interventions to decrease
the anxiety of hospitalization.
(C) Repeating the physician's reason for recommending hospitalization may not aid the client in dealing
with her reasons for anxiety.
(D) The concern for self and welfare of baby may be secondary to a woman who is in a crisis situation.
The nurse should explore the client's potential reasons for anxiety. For example, is there another child in
the home who is ill, or is there a husband who is overseas and not able to return on short notice?
405. A male client has asthma and his physician has prescribed beclomethasone (Vanceril) 3 puffs tid in
addition to his other medications. After taking his beclomethasone, the client should be instructed to:
A. Clean his inhaler with warm water and soak it in a 10% bleach solution
B. Drink a glass of water
C. Sit and rest
D. Use his bronchodilator inhaler
Answer: B
Explanation:
(A) Inhalers should be cleaned once a day. They should be taken apart, washed in warm water, and dried
according to manufacturer's instructions. Soaking in bleach is inappropriate.
(B) A common side effect of inhaled steroid preparations is oral candidal infection. This can be prevented
by drinking a glass of water or gargling after using a steroid inhaler.
(C) There is nothing wrong with sitting and resting after using a steroid inhaler, but it is not necessary.
(D) If a person is using a steroid inhaler as well as a bronchodilator inhaler, the bronchodilator should
always be used first. The reason for this is that the bronchodilator opens up the person's airways so that
when the steroid inhaler is used next, there will be better distribution of medication.
406. An obstructing stone in the renal pelvis or upper ureter causes:
A. Radiating pain into the urethra with labia pain experienced in females or testicular pain in males

B. Urinary frequency and dysuria
C. Severe flank and abdominal pain with nausea, vomiting, diaphoresis, and pallor
D. Dull, aching, back pain
Answer: C
Explanation:
(A) Radiating pain in the urethra in both sexes, extending into the labia in females and into the testicle or
penis in the male, indicates a stone in the middle or lower segment of the ureter.
(B) Urinary frequency and dysuria are caused by a stone in the terminal segment of the ureter withinthe
bladder wall.
(C) An obstructing stone in the renal pelvis or upper ureter causes severe flank and abdominal pain with
nausea, vomiting, diaphoresis, and pallor.
(D) Dull and aching pain may indicate early stages of hydronephrosis. Also, a stone in the renal pelvis or
upper ureter causes severe flank and abdominal pain.
407. A first-trimester primigravida is diagnosed with anemia.
The nurse should suspect that this anemia is a result of:
A. Mother's increased blood volume
B. Mother's decreased blood volume
C. Fetal blood volume increase
D. Increase in iron absorption
Answer: A
Explanation:
(A) Maternal blood volume increases at the end of the first trimester leading to a dilutional anemia.
(B) Maternal blood volume increases.
(C) Fetal blood volume is minimal in the first trimester.
(D) Increased iron absorption would facilitate the manufacturing of erythrocytes and decrease anemia.
408. The nurse is teaching a mother care of her child's spica cast. The mother states that he complains of
itching under the edge of the cast. One nonpharmacological technique the nurse might suggest would be:
A. "Blowing air under the cast using a hair dryer on cool setting often relieves itching."
B. "Slide a ruler under the cast and scratch the area."
C. "Guide a towel under and through the cast and move it back and forth to relieve the itch."
D. "Gently thump on cast to dislodge dried skin that causes the itching."

Answer: A
Explanation:
(A) Cool air will often relieve pruritus without damaging the cast or irritating the skin.
(B) The nurse should never force anything under the cast, because the cast may become damaged and skin
breakdown may occur.
(C) Forcing an object under the cast could lead to cast damage and skin breakdown. The object may
become lodged under the cast necessitating cast removal.
(D) This technique does not dislodge skin cells. It could damage the cast and cause skin breakdown.
409. A client is resting comfortably after delivering her first child. When assessing her pulse rate, the nurse
would recognize the following finding to be typical:
A. Thready pulse
B. Irregular pulse
C. Tachycardia
D. Bradycardia
Answer: D
Explanation:
(A) A thready pulse is indicative of hypotension and excessive blood loss and is often rapid.
(B) Pulse irregularities or dysrhythmias do not occur in the normal post partal woman.
(C) Tachycardia occurs less frequently than bradycardia and is related to increased blood loss or prolonged
difficult labor and/or birth.
(D) Puerperal bradycardia with rates of 50-70 bpm commonly occurs during the first 6-10 days of the post
partal period. It may be related to decreased cardiac strain, decreased blood volume, contraction of the
uterus, and increased stroke volume.
410. When interviewing parents who are suspected of child abuse, the nurse would use which of the
following interview techniques?
A. Be direct, honest, and attentive.
B. Approach them in the emergency room as soon as you suspect abuse to "clear the air" right away.
C. Ask the parents what they could have done differently to prevent this from happening to the child.
D. After the interview, call child protective services.
Answer: A
Explanation:

(A) The nurse must be honest, direct, professional, and attentive in her interview to gain the parent's trust.
(B) The nurse should approach the parents in private, away from the child.
(C) Asking them to relive and evaluate the situation may be looked at as placing blame on the parents for
the child's "accident." At this point, the parents may get defensive and stop communicating.
(D) Although you may call child protective services, the nurse should inform the parents of their
responsibility to do this and explain the process to them.
411. A pregnant woman at 36 weeks' gestation is followed for PIH and develops proteinuria. To increase
protein in her diet, which of the following foods will provide the greatest amount of protein when added to
her intake of 100 mL of milk?
A. Fifty milliliters light cream and 2 tbsp corn syrup
B. Thirty grams powdered skim milk and 1 egg
C. One small scoop (90 g) vanilla ice cream and 1 tbsp chocolate syrup
D. One package vitamin-fortified gelatine drink
Answer: B
Explanation:
(A) This choice would provide more unwanted fat and sugar than protein.
(B) Skim milk would add protein. Eggs are good sources of protein while low in fat and calories.
(C) The benefit of protein from ice cream would be outweighed by the fat content. Chocolate syrup has
caffeine, which is contraindicated or limited in pregnancy.
(D) Although most animal proteins are higher in protein than plant proteins, gelatin is not. It loses protein
during the processing for food consumption.
412. The nurse teaches a pregnant client that a high-risk symptom occurring at any time during pregnancy
that needs to be reported immediately to a healthcare provider is:
A. Constipation
B. Urinary frequency
C. Breast tenderness
D. Abdominal pain
Answer: D
Explanation:
(A) Constipation is a result of decreased peristalsis due to smooth muscle relaxation related to changing
progesterone levels that occur during pregnancy.

(B) Urinary frequency is a common result of the increasing size of the uterus and the resulting pressure it
places on the bladder.
(C) With the increased vascularity and hypertrophy of the mammary alveoli due to estrogen and
progesterone level changes, the breasts will increase in size and may become tender.
(D) Abdominal pain may be an indication of early spontaneous abortion, preterm delivery, or a placental
abruption.
413. A 15-year-old female adolescent is frequently breaking the rules of the unit. She has left the unit and
was found smoking in the bathroom and spending a large amount of time in the male ward. Which
statement by the nurse would best explain to the teenager why she must follow the rules of the unit?
A. "It is not easy, but the rules must be followed so that everyone can get a fair chance."
B. "If you do not follow the rules, you will be transferred to the closed, locked unit."
C. "You are not being fair to the other clients by getting them involved in your deviant behavior."
D. "Break the rules, all you want, but don't get caught again!"
Answer: A
Explanation:
(A) This statement acknowledges that it is difficult but is not threatening or punitive.
(B) This statement is threatening and describes specific punishment for further deviant behavior.
(C) This response elicits shame by blaming her for involving others.
(D) This response gives her permission to break the rules but indicates that getting caught is wrong.
414. A 28-year-old woman was admitted to the hospital for a thyroidectomy. Postoperatively she is taken
to the post anesthesia care unit for several hours. In preparing for the client's return to her room, which
nursing measure best demonstrates the nurse's thorough understanding of possible post thyroidectomy
complications?
A. Dressings are placed at the bedside for dressing changes, which are to be done every 2 hours to best
detect postoperative bleeding.
B. Narcotics are readily available and administered when the client returns to her room to prevent
excruciating pain.
C. A tracheostomy set, O2, and suction are available at the bedside.
D. The nurse should instruct the client as soon as possible on alternative means of communication.
Answer: C
Explanation:

(A) Dressing changes are done as necessary for bleeding. However, frequently, post-thyroidectomy
bleeding may not be visible on the dressing, but blood may drain down the back of the neck by gravity.
(B) Narcotics are administered for acute pain as necessary. They are not necessarily given on return of the
client to her room.
(C) The most serious post thyroidectomy complication is ineffective airway and breathing pattern related
to tracheal compression and edema. A tracheostomy set, O2, and suction should be available at bedside for
at least the first 24 hours postoperatively.
(D) Impaired verbal communication may occur due to laryngeal edema or nerve damage, but most
commonly, it occurs due to endotracheal intubation. The client is usually able to communicate but is
hoarse.
415. A 6-year-old girl is visiting the outpatient clinic because she has a fever and a rash. The doctor
diagnoses chickenpox. Her mother asks the nurse how many baby aspirins her daughter can have for fever.
The nurse should:
A. Advise the mother not to give her aspirin
B. Ask if the client is allergic to aspirin before giving further information
C. Assess the function of the client's cranial nerve VIII
D. Check the aspirin bottle label to determine milligrams per tablet
Answer: A
Explanation:
(A) Aspirin taken during a viral infection has been implicated as a predisposing factor to Reye's syndrome
in children and adolescents. Children and adolescents should not be given aspirin.
(B) Allergy to aspirin is not related to Reye's syndrome.
(C) Tinnitus, caused by damage to the acoustic nerve, occurs with aspirin toxicity, but this is not related to
Reye's syndrome.
(D) A 6-year-old child should not be given any baby aspirin.
416. A 50-year-old male client is to receive chemotherapy. The physician's orders include antiemetics.
When planning his care, the nurse should take into consideration that antiemetics are best administered in
the following way:
A. Give antiemetics when nausea is experienced and continue on a regular schedule for 12-24 hours.
B. Give antiemetics prior to the client receiving chemotherapy and continue on a regular basis for at least
24-48 hours after chemotherapy.

C. Give antiemetics one at a time because combinations of antiemetics cause overwhelming side effects.
D. Give antiemetics intermittently during the entire course of chemotherapy.
Answer: B
Explanation:
(A) Nausea is more difficult to control if antiemetics are withheld until nausea is experienced.
(B) Antiemetics should be given prophylactically at the beginning of chemotherapy and continued on an
around-the-clock basis to prevent nausea.
(C) Combinations of antiemetics give the best control for nausea by blocking various causes of nausea
induced by chemotherapy.
(D) Antiemetics should be given around the clock during the course of chemotherapy. This prevents nausea
from developing and prevents anticipatory nausea during subsequent chemotherapy administrations.
417. The nurse practitioner determines that a client is approximately 9 weeks' gestation. During the visit,
the practitioner informs the client about symptoms of physical changes that she will experience during her
first trimester, such as:
A. Nausea and vomiting
B. Quickening
C. A 6-8 lb weight gain
D. Abdominal enlargement
Answer: A
Explanation:
(A) Nausea and vomiting are experienced by almost half of all pregnant women during the first 3 months
of pregnancy as a result of elevated human chorionic gonadotropin levels and changed carbohydrate
metabolism.
(B) Quickening is the mother's perception of fetal movement and generally does not occur until 18-20
weeks after the last menstrual period in primigravidas, but it may occur as early as 16 weeks in
multigravidas.
(C) During the first trimester there should be only a modest weight gain of 2-4 lb. It is not uncommon for
women to lose weight during the first trimester owing to nausea and/or vomiting.
(D) Physical changes are not apparent until the second trimester, when the uterus rises out of the pelvis.
418. A client with severe PIH receiving MgSO4 is placed in a quiet, darkened room. The nurse bases this
action on the following understanding:

A. The client is restless.
B. The elevated blood pressure causes photophobia.
C. Noise or bright lights may precipitate a convulsion.
D. External stimuli are annoying to the client with PIH.
Answer: C
Explanation:
(A) The client may be anxious and hyperresponsive to stimuli but not necessarily restless.
(B) This is not a physiological response to an elevated blood pressure in PIH.
(C) The nurse must know the nursing measures that decrease the potential for convulsions. A quiet,
darkened room decreases stimuli and promotes rest.
(D) External stimuli might induce a convulsion but are not annoying to the client with PIH.
419. A client has consented to have a central venous catheter placed. The best position in which to place
the client is the Trendelenburg position. The reason is that the Trendelenburg position:
A. Allows the physician to visualize the subclavian vein
B. Reduces the possibility of air embolism
C. Reduces the possibility of hematoma formation
D. Makes the procedure more comfortable for the client
Answer: B
Explanation:
(A) The subclavian vein is not visible during central line insertion regardless of the client's position.
(B) The Trendelenburg position reduces the possibility of air embolism because it places slight positive
pressure on the central veins. It also distends the veins, and distention facilitates insertion.
(C) This response is untrue; it has no effect on hematoma formation.
(D) This position is not necessarily more comfortable for the client, and many clients, especially those who
may be short of breath, may find the position uncomfortable and difficult to maintain.
420. A newborn infant is exhibiting signs of respiratory distress. Which of the following would the nurse
recognize as the earliest clinical sign of respiratory distress?
A. Cyanosis
B. Increased respirations
C. Sternal and subcostal retractions
D. Decreased respirations

Answer: C
Explanation:
(A) Cyanosis is a late clinical sign of respiratory distress.
(B) Rapid respirations are normal in a newborn.
(C) The newborn has to exert an extra effort for ventilation, which is accomplished by using the accessory
muscles of ventilation. The diaphragm and abdominal muscles are immature and weak in the newborn.
(D) Decreased respirations are a late clinical sign. In the newborn, decreased respirations precede
respiratory failure.
421. A 32-year-old male client is a marketing representative. His job requires him to have a tremendous
amount of energy during the day. He frequently uses cocaine to sustain his energy level. Lately he has
increased his use of cocaine and even experimented with crack cocaine. Realizing he can no longer
continue this destructive behavior, he is seeking treatment for cocaine addiction. In planning nursing care
for the client's inpatient stay, which expected outcome is most appropriate?
A. He will attend four consecutive group educational sessions on substance abuse.
B. He will name activities that he would most likely be involved in posttreatment.
C. He will meet with his family in counseling sessions and discuss his feelings.
D. He will be able to deal with his feelings through participation in group therapy sessions.
Answer: D
Explanation:
(A) This expected outcome is specific as related to attendance, but not specific as related to outcome
criteria.
(B) Stating activities does not guarantee involvement.
(C) This goal may help the recovery process, but post counseling behavior is not addressed.
(D) This statement best describes the expected outcome. The client will be attending group therapy
sessions and through them he will deal with his feelings.
422. The physician prescribes amitriptyline (Elavil) for a client. What does the patient need to know about
this medication?
A. Prolonged use of this medication will result in extrapyramidal side effects.
B. When the medication is effective, he will experience no anxiety.
C. The medication should relieve his symptoms of depression.
D. Blood must be drawn weekly to test for toxicity.

Answer: C
Explanation:
(A) Phenothiazines cause extrapyramidal symptoms.
(B) No amount of medication can relieve all anxiety in all cases.
(C) The purpose of amitriptyline is to relieve the symptoms of depression because it is an antidepressant. It
increases the action of norepinephrine and serotonin on nerve cells.
(D) Periodic blood tests are done when lithium is prescribed.
423. A registered nurse is trying to determine the appropriate care that she should provide for her
obstetrical clients. Which of the following documents is considered the legal standard of practice?
A. State nursing practice act
B. AWHONN Standards for the Nursing Care of Women and Newborns
C. American Nurses' Association Standards of Maternal- Child Health Nursing
D. International Council of Nurses' Code
Answer: A
Explanation:
(A) The state nursing practice act determines the standard of care for the professional nurse.
(B) AWHONN Standards are published as recommendations and guidelines for maternal-newborn nursing.
(C) American Nurses' Association Standards are published as recommendations and guidelines for
maternal child health nursing.
(D) The International Council of Nurses' Code emphasizes the nurse's obligations to the client rather than
to the physician. It is published as recommendations and guidelines by the international organization for
professional nursing.
424. A 55-year-old woman entered the emergency room by ambulance. Her primary complaint is chest
pain. She is receiving O2 via nasal cannula at 2 L/min for dyspnea. Which of the following findings in the
client's nursing assessment demand immediate nursing action?
A. Associated symptoms of indigestion and nausea
B. Restlessness and apprehensiveness
C. Inability to tolerate assessment session with the admitting nurse
D. History of hypertension treated with pharmacological therapy
Answer: B
Explanation:

(A) Indigestion or nausea may accompany angina or myocardial infarction, but they do not indicate
imminent danger for the client.
(B) Restlessness and apprehensiveness require immediate nursing action because they are indicative of
very low oxygenation of body tissues and are frequently the first indication of impending cardiac or
respiratory arrest.
(C) It is common for the cardiac client to experience fatigue and inability to physically tolerate long
assessment sessions.
(D) A history of hypertension requires no immediate nursing intervention. In the situation described, the
blood pressure is not given and therefore cannot be assumed to be elevated.
425. Parents of young children often need anticipatory guidance from the nurse. Parents may have little
knowledge regarding growth and development. Which of the following toys and activities would the nurse
suggest as appropriate for a toddler?
A. Cutting, pasting, string beads, music, dolls
B. Mobiles, rattle, squeeze toys
C. Pull-toys, large ball, dolls, sand and water play, music
D. Simple card games, puzzles, bicycle, television
Answer: C
Explanation:
(A) These activities are suited for the preschool-age child (3-5 years old). The activities are not safe for a
toddler.
(B) Infants (0-1 year) like these toys.
(C) These activities provide the toddler (1-3 years old) with a variety of physical activities for play.
(D) The toddler lacks the physical and cognitive abilities for these activities. The tasks are far better suited
for the school-age child.
426. A 23-year-old female client is brought to the emergency room by her roommate for repeatedly making
superficial cuts on her wrists and experiencing wide mood swings. She is very angry and hostile. Her
medical diagnosis is adjustment disorder versus borderline personality disorder. The client comments to
the nurse, "Nobody in here seems to really care about the clients. I thought nurses cared about people!"
The client is exhibiting the ego defense mechanism:
A. Reaction formation
B. Rationalization

C. Splitting
D. Sublimation
Answer: C
Explanation:
(A) Reaction formation is the development and demonstration of attitudes and/or behaviors opposite to
what an individual actually feels. The client's comment does reveal her anger and hostility.
(B) Rationalization, another ego defense mechanism, is offering a socially acceptable or seemingly logical
explanation to justify one's feelings, behaviors, or motives. The client's comment does not reflect
rationalization.
(C) Splitting, the viewing of people or situations as either all good or all bad, is frequently used by persons
experiencing a disruption in self-concept. This ego defense mechanism is reflective of the individual's
inability to integrate the positive and negative aspects of self.
(D) Sublimation, the channeling of socially unacceptable impulses and behaviors into more acceptable
patterns of behavior, is another ego defense mechanism. The client's comment reveals that she is not
engaging in sublimation.
427. A female client has married recently. A month ago she visited her physician with complaints of
burning on urination. She was given a prescription for trimethoprim- sulfamethoxazole (Bactrim) DS bid
for 10 days. She was admitted through the emergency room on Saturday evening complaining of flank
pain. Her temperature was 104_F. A preliminary urinalysis revealed 31 bacteria along with red and white
blood cells in the urine. A preliminary diagnosis of pyelonephritis was made. During a nursing admission
assessment, which statement by the client demonstrates a possible cause for pyelonephritis?
A. "I have not been drinking six to eight glasses of water each day as the nurse had instructed."
B. "I'm afraid I may have something wrong with my bladder because I have been getting bladder infections
frequently since I've been married."
C. "I took the Bactrim for 6 or 7 days. The burning stopped, so I saved the rest of the medication for the
next time."
D. "I recently had the flu, which could be settling in my kidneys now."
Answer: C
Explanation:
(A) Although it is important that the client drink adequate fluids while treating a bladder infection with
trimethoprim sulfamethoxazole, the failure to do so will not cause pyelonephritis.

(B) A stricture or abnormality may cause the progression of bladder infection to urinary tract infection, but
this is rare. There is no indication in this situation that this has occurred.
(C) The most common cause of pyelonephritis is improper treatment of bladder infections. The client
typically feels better after several days, discontinues the medication, and saves the remainder forthe next
occurrence of a bladder infection. For this reason, it is imperative to provide client education related to
completion of the prescribed medication.
(D) There is no evidence that infection in another body system could cause pyelonephritis.
428. Morphine sulfate 4 mg IV push q2h prn for chest pain was ordered for a client in the emergency room
with severe chest pain. The nurse administering the morphine sulfate knows which of the following
therapeutic actions is related to the morphine sulfate?
A. Increased level of consciousness
B. Increased rate and depth of respirations
C. Increased peripheral vasodilation
D. Increased perception of pain
Answer: C
Explanation:
(A) Morphine sulfate, a narcotic analgesic, causes sedation and a decrease in level of consciousness.
(B) The side effects of morphine sulfate include respiratory depression.
(C) Morphine sulfate causes peripheral vasodilation, which decreases afterload, producing a decrease in
the myocardial workload.
(D) Morphine sulfate alters the perception of pain through an unclear mechanism. This alteration promotes
pain relief.
429. An 82-year-old former restaurant owner walks to the nursing station and states, "I have to go. The
restaurant opens at 11 am." Which response by the nurse is the most appropriate?
A. "Go back to your room. You do not own a restaurant."
B. "You are in the hospital now. Calm down."
C. "You once owned a restaurant. Tell me about it."
D. "It is snowing outside. The restaurant is closed."
Answer: C
Explanation:
(A) This response cuts off communication with the client. It does not address her feelings.

(B) Reality orientation frequently does not work alone. Feelings must be addressed. Telling a client to calm
down is frequently ineffective.
(C) Reminiscence is used here to reorient and recall past pleasant events. Talking about the restaurant will
allay anxiety.
(D) This response may confirm to the client that she indeed does still own a restaurant, buying into her
confusion. Her feelings and anxiety require nursing intervention.
430. A male infant is to be discharged home this morning. Which instruction related to his cord care should
be included in his mother's discharge teaching plan?
A. Keep the umbilical area moist with Vaseline until the stump falls off.
B. Keep the umbilical area covered at all times with the diaper.
C. Clean the umbilical cord with alcohol at each diaper change.
D. Clean the umbilical cord daily with soap and water during the bath.
Answer: C
Explanation:
(A) The umbilical area should be kept dry for healing to occur. Moisture is conducive to bacterial growth
and therefore could lead to infection at the site.
(B) The diaper should be folded below the cord to allow the cord stump to be exposed to the air for
healing.
(C) The umbilical cord should be swabbed with alcohol at each diaper change to remove urine and stool
and to facilitate the desiccation process through drying.
(D) Soap and water should not be used to clean the umbilical area because the area could retain moisture,
thus making it susceptible to bacterial growth and infection.
431. Iron dextran (Imferon) is a parenteral iron preparation. The nurse should know that it:
A. Is also called intrinsic factor
B. Must be given in the abdomen
C. Requires use of the Z-track method
D. Should be given SC
Answer: C
Explanation:
(A) Intrinsic factor is needed to absorb vitamin B12.(B) Iron dextran is given parenterally, but Z-track in a
large muscle.

(C) A Ztrack method of injection is required to prevent staining and irritation of the tissue.
(D) An SC injection is not deep enough and may cause subcutaneous fat abscess formation.
432. A client develops complications following a hysterectomy. Blood cultures reveal Pseudomonas
aeruginosa. The nurse expects that the physician would order an appropriate antibiotic to treat Paeruginosa
such as:
A. Cefoperazone (Cefobid)
B. Clindamycin (Cleocin)
C. Dicloxacillin (Dycill)
D. Erythromycin (Erythrocin)
Answer: A
Explanation:
(A) Cefoperazone is indicated in the treatment of infection with Pseudomonas aeruginosa.
(B) Clindamycin is not indicated in the treatment of infection with Paeruginosa.
(C) Dicloxacillin is not indicated in the treatment of infection with Paeruginosa.
(D) Erythromycin is not indicated in the treatment of infection with Paeruginosa.
433. A 74-year-old client seen in the emergency room is exhibiting signs of delirium. His family states that
he has not slept, eaten, or taken fluids for the past 24 hours. The planning of nursing care for a delirious
client is based on which of the following premises?
A. The delirious client is capable of returning to his previous level of functioning.
B. The delirious client is incapable of returning to his previous level of functioning.
C. Delirium entails progressive intellectual and behavioral deterioration.
D. Delirium is an insidious process.
Answer: A
Explanation:
(A) This answer is correct. If the cause is removed, the delirious client will recover completely.
(B) This answer is incorrect. The demented client is incapable of returning to previous level of functioning.
The delirious client is capable of returning to previous functioning.
(C) This answer is incorrect. The demented client, not the delirious client, has progressive intellectual and
behavioral deterioration.
(D) This answer is incorrect. Delirium develops rapidly, whereas dementia is insidious.

434. A client on the infectious disease unit is discussing transmission of human immunodeficiency virus
(HIV).
The nurse would need to provide more client education based on which client statement?
A. "HIV is a virus transmitted by sexual contact."
B. "Condoms reduce the transmission of HIV."
C. "HIV is a virus that is easily transmitted by casual contact."
D. "HIV can be transmitted to an unborn infant."
Answer: C
Explanation:
(A) HIV is transmitted through unprotected sexual contact.
(B) Condoms are an effective barrier to prevent HIV transmission.
(C) HIV is not easily transmitted by casual contact.
(D) HIV can be transmitted in trauterinely at the time of delivery, and by breast-feeding.
435. A 2-month-old infant is receiving IV fluids with a volume control set. The nurse uses this type of
tubing because it:
A. Prevents administration of other drugs
B. Prevents entry of air into tubing
C. Prevents inadvertent administration of a large amount of fluids
D. Prevents phlebitis
Answer: C
Explanation:
(A) A volume control set has a chamber that permits the administration of compatible drugs.
(B) Air may enter a volume control set when tubing is not adequately purged.
(C) A volume control set allows the nurse to control the amount of fluid administered over a set period.
(D) Contamination of volume control set may cause phlebitis.
436. In addition to changing the mother's position to relieve cord pressure, the nurse may employ the
following measure (s) in the event that she observes the cord out of the vagina:
A. Immediately pour sterile saline on the cord, and repeat this every 15 minutes to prevent drying.
B. Cover the cord with a wet sponge.
C. Apply a cord clamp to the exposed cord, and cover with a sterile towel.
D. Keep the cord warm and moist by continuous applications of warm, sterile saline compresses.

Answer: D
Explanation:
(A) Saline should be warmed; waiting 15 minutes may not keep the cord moist.
(B) This choice does not specify what the sponge was "wet" with.
(C) This measure would stop circulation to the fetus.
(D) The cord should be kept warm and moist to maintain fetal circulation. This measure is an accepted
nursing action.
437. A pregnant client is at the clinic for a third trimester prenatal visit. During this examination, it has
been determined that her fetus is in a vertex presentation with the occiput located in her right anterior
quadrant. On her chart this would be noted as:
A. Right occipito posterior
B. Right occipito anterior
C. Right sacro anterior
D. LOA
Answer: B
Explanation:
(A) The fetus in the right occipito posterior position would be presenting with the occiput in the maternal
right posterior quadrant.
(B) Fetal position is defined by the location of the fetal presenting part in the four quadrants of the
maternal pelvis. The right occipito anterior is a fetus presenting with the occiput in mother's right anterior
quadrant.
(C) The fetus in right sacro anterior position would be presenting a sacrum, not an occiput.
(D) The fetus in left occipito anterior position would be presenting with the occiput in the mother's left
anterior quadrant.
438. A client who is a breast-feeding mother develops mastitis. The clinical signs and symptoms of mastitis
include:
A. Marked engorgement, elevated temperature, chills, and breast pain with an area that is red and hardened
B. Marked engorgement and breast pain
C. Elevated temperature and general malaise
D. Cracked nipple with complaints of soreness
Answer: A

Explanation:
(A) Mastitis is a bacterial inflammation of the breast tissue found primarily in breast-feeding mothers. The
bacteria usually enter the breast through a cracked nipple, or the infection results from stasis of milk
behind a blocked duct.
(B) With breast engorgement during breast-feeding, there may be marked breast pain. This is not
necessarily a sign of infection.
(C)Women may become ill during breast-feeding with other bacterial or viral infections that are not related
to mastitis.
(D) Improper care of the nipples or improper positioning of the infant during breastfeeding may result in
cracked or sore nipples.
439. A 26-year-old client is admitted to the labor, delivery, recovery, postpartum unit. The nurse completes
her assessment and determines the client is in the first stage of labor. The nurse should instruct her:
A. To hold her breath during contractions
B. To be flat on her back
C. Not to push with her contractions
D. To push before becoming fully dilated
Answer: C
Explanation:
(A) This nursing action may cause hyperventilation.
(B) This nursing action could cause inferior vena cava syndrome.
(C) The client is allowed to push only after complete dilation during the second stage of labor. The nurse
needs to know the stages of labor.
(D) If the client pushes before dilation, it could cause cervical edema and/or edema to the fetal scalp; both
of these could contribute to increased risk of complications.
440. In a client with chest trauma, the nurse needs to evaluate mediastinal position. This can best be done
by:
A. Auscultating bilateral breath sounds
B. Palpating for presence of crepitus
C. Palpating for trachial deviation
D. Auscultating heart sounds
Answer: C

Explanation:
(A) No change in the breath sounds occurs as a direct result of the mediastinal shift.
(B) Crepitus can occur owing to the primary disorder, not to the mediastinal shift.
(C) Mediastinal shift occurs primarily with tension pneumothorax, but it can occur with very large
hemothorax or pneumothorax.
Mediastinal shift causes trachial deviation and deviation of the heart's point of maximum impulse.
(D) No change in the heart sounds occurs as a result of the mediastinal shift.
441. A 68-year-old man was recently diagnosed with end stage renal disease. He has not yet begun dialysis
but is experiencing severe anemia with associated symptoms of dyspnea on exertion and chest pain. Which
statement best describes the management of anemia in renal failure?
A. Haematocrit levels usually remain slightly below normal in clients with renal failure.
B. Transfusion is often begun as early as possible to prevent complications of anemia such as dyspnea and
angina.
C. Anemia in renal failure is frequently caused by low serum iron and ferritin and corrected by oral iron
and ferritin replacement therapy.
D. The renal secretion of erythropoiesis is decreased. The bone marrow requires erythropoietin to mature
red blood cells.
Answer: D
Explanation:
(A) Clients in renal failure typically have very low haematocrits, often in the range of 16-22%.
(B) Transfusion is avoided unless the client exhibits acute symptoms such as dyspnea, chest pain,
tachycardia, and extreme fatigue. When the client is given a transfusion, the bone marrow adjusts by
producing less red blood cells.
(C) Anemia in renal failure is caused primarily by decreased erythropoietin. Low serum iron and ferritin
may aggravate the anemia and require treatment.
(D) Decreased secretion of erythropoietin by the kidney is the primary cause of anemia. The bone marrow
requires this hormone to mature red blood cells. Treatment is with replacement therapy.
442. A 10-year-old has been diagnosed with acute poststreptococcal glomerulonephritis. The clinical
findings were proteinuria, moderately elevated blood pressure, and periorbital edema. Which dietary plan
is most appropriate for this client?
A. Low-protein diet

B. Low-sodium diet
C. Increased fluid intake
D. High-cholesterol diet
Answer: B
Explanation:
(A) A high-protein diet is usually indicated because protein is excreted in urine. Protein restriction is
usually prescribed with severe azotemia.
(B) The kidneys usually enlarge in these children, and sodium and water are retained.
(C) Fluid restriction may be ordered to help reduce edema; however, monitoring for dehydration is
indicated.
(D) A high-cholesterol diet would not be indicated for any child, especially one with elevated blood
pressure.
443. In performing the initial nursing assessment on a client at the prenatal clinic, the nurse will know that
which of the following alterations is abnormal during pregnancy?
A. Striae gravidarum
B. Chloasma
C. Dysuria
D. Colostrum
Answer: C
Explanation:
(A) Striae gravidarum are the normal stretch marks that frequently occur on the breasts, abdomen, and
thighs as pregnancy progresses.
(B) Chloasma is the "mask of pregnancy" that normally occurs in many pregnant women.
(C) Dysuria is an abnormal danger sign during pregnancy and may indicate a urinary tract infection.
(D) Colostrum is a yellow breast secretion that is normally present during the last trimester of pregnancy.
444. Which classification of drugs is contraindicated for the client with hypertrophic cardiomyopathy?
A. Positive inotropes
B. Vasodilators
C. Diuretics
D. Antidysrhythmic
Answer: A

Explanation:
(A) Positive inotropic agents should not be administered owing to their action of increasing myocardial
contractility. Increased ventricular contractility would increase outflow tract obstruction in the client with
hypertrophic cardiomyopathy.
(B) Vasodilators are not typically prescribed but are not contraindicated.
(C) Diuretics are used with caution to avoid causing hypovolemia.
(D) Antidysrhythmic are typically needed to treat both atrial and ventricular dysrhythmias.
445. A client has been diagnosed with congestive heart failure. His fluid intake and output are strictly
regulated. For lunch, he drank 8 oz of milk, 4 oz of tea, and 6 oz of coffee. His intake would be recorded as:
A. 500 mL
B. 540 mL
C. 600 mL
D. 655 mL
Answer: B
Explanation:
(A, C, D) This answer is a miscalculation.
(B) 1 oz = 30 mL; therefore, 18 oz x.
446. Painless vaginal bleeding in the last trimester may be caused by:
A. Menstruation
B. Abruptio placentae
C. Placenta previa
D. Polyhydramnios
Answer: C
Explanation:
(A) Menstruation should not occur during pregnancy.
(B) Abruptio placentae is marked by painful vaginal bleeding following a premature placental detachment
after 20th week of gestation.
(C) A low lying placenta separates from the uterine wall as the uterus contracts and cervix dilates. This
separation causes painless bleeding in the 7th-8th month.
(D) Polyhydramnios is excessive amniotic fluid.

447. The nurse should know that according to current thinking, the most important prognostic factor for a
client with breast cancer is:
A. Tumor size
B. Axillary node status
C. Client's previous history of disease
D. Client's level of estrogen-progesterone receptor assays
Answer: B
Explanation:
(A) Although tumor size is a factor in classification of cancer growth, it is not an indicator of lymph node
spread.
(B) Axillary node status is the most important indicator for predicting how far the cancer has spread. If the
lymph nodes are positive for cancer cells, the prognosis is poorer.
(C) The client's previous history of cancer puts her at an increased risk for breast cancer recurrence,
especially if the cancer occurred in the other breast. It does not predict prognosis, however.
(D) The estrogen progesterone assay test is used to identify present tumors being fed from an estrogen site
within the body. Some breast cancers grow rapidly as long as there is an estrogen supply such as from the
ovaries. The estrogen-progesterone assay test does not indicate the prognosis.
448. The primary reason that an increase in heart rate (100 bpm) detrimental to the client with a
myocardial infarction (MI) is that:
A. Stroke volume and blood pressure will drop proportionately
B. Systolic ejection time will decrease, thereby decreasing cardiac output
C. Decreased contractile strength will occur due to decreased filling time
D. Decreased coronary artery perfusion due to decreased diastolic filling time will occur, which will
increase ischemic damage to the myocardium
Answer: D
Explanation:
(A) Decreased stroke volume and blood pressure will occur secondary to decreased diastolic filling.
(B) Tachycardia primarily decreases diastole; systolic time changes very little.
(C) Contractility decreases owing to the decreased filling time and decreased time for fiber lengthening.
(D) Decreased O2 supply due to decreased time for filling of the coronary arteries increases ischemia and
infarct size. Tachycardia primarily robs the heart of diastolic time, which is the primary time for coronary
artery filling.

449. Assessment of a client reveals a 30% loss of pre-illness weight, lanugo, and cessation of menses for 3
months. Her vital signs are BP 90/50, P 96 bpm, respirations 30, and temperature 97 F. She admits to the
nurse that she has induced vomiting 3 times this morning, but she had to continue exercising to lose "just 5
more lb." Her symptoms are consistent with:
A. Pregnancy
B. Bulimia
C. Gastritis
D. Anorexia nervosa
Answer: D
Explanation:
(A) Presenting behaviors collectively are inconsistent with depression.
(B) A pre-illness weight loss of 30%, lanugo, and cessation of menses are inconsistent with bulimia.
(C) Symptoms and vital signs do not indicate the presence of infection.
(D) All symptoms and vital signs are consistent with anorexia nervosa.
450. A 14-year-old boy fell off his bike while "popping a wheelie" on the dirt trails. He has sustained a
head injury with laceration of his scalp over his temporal lobe. If he were to complain of headache during
the first 24 hours of his hospitalization, the nurse would:
A. Ask the physician to order a sedative
B. Have the client describe his headache every 15 minutes
C. Increase his fluid intake to 3000 mL/24 hr
D. Offer diversionary activities
Answer: D
Explanation:
(A) CNS depressants are not given for headache due to head injury because they would mask changes in
neurological status and because they could further depress the CNS.
(B) The client should not be asked to think about his headache every 15 minutes.
(C) Fluid intake should be normal or restricted for a client with a head injury. Normal fluid intake for a 14
year old is about 2000-2400 mL daily.
(D) Diversion may help the child to focus on a pleasant activity instead of on his headache.

451. A female client is admitted to the emergency department complaining of severe right-sided abdominal
pain and vaginal spotting. She states that her last menstrual period was about 2 months ago. A positive
pregnancy test result and ultrasonography confirm an ectopic pregnancy. The nurse could best explain to
the client that her condition is caused by:
A. Abnormal development of the embryo
B. A distended or ruptured fallopian tube
C. A congenital abnormality of the tube
D. A malfunctioning of the placenta
Answer: B
Explanation:
(A) The embryo itself may develop normally in the first several weeks of an ectopic pregnancy.
(B) An ectopic pregnancy in the fallopian tube causes severe pain owing to the size of the growing embryo
within the narrow lumen of the tube, causing distention and finally rupture within the first 12 weeks of
pregnancy.
(C) The Fallopian tube may either be normal or contain adhesions caused by a history of pelvic
inflammatory disease or tubal surgeries, neither of which are congenital causes.
(D) An ectopic pregnancy does not involve a dysfunctional placenta, but the implantation of the blastocyst
outside the uterus.
452. A chronic alcoholic client's condition deteriorates, and he begins to exhibit signs of hepatic coma.
Which of the following is an early sign of impending hepatic coma?
A. Hiccups
B. Anorexia
C. Mental confusion
D. Fetor hepaticus
Answer: C
Explanation:
(A) Hiccups are not a sign of impending hepatic coma.
(B) Anorexia is not a sign of impending hepatic coma.
(C) One of the earliest symptoms of hepatic coma is mental confusion. Asterixis, a flapping tremor of the
hand, may also be seen.
(D) This sign is associated with the later stages of hepatic coma. Fetor hepaticus, a characteristic odor on
the breath that smells like acetone, may sometimes be noted when the liver fails.

453. A mother continues to breast-feed her 3-month-old infant. She tells the nurse that over the past 3 days
she has not been producing enough milk to satisfy the infant. The nurse advises the mother to do which of
the following?
A. "Start the child on solid food."
B. "Nurse the child more frequently during this growth spurt."
C. "Provide supplements for the child between breastfeeding so you will have enough milk."
D. "Wait 4 hours between feedings so that your breasts will fill up."
Answer: B
Explanation:
(A) Solid foods introduced before 4-6 months of age are not compatible with the abilities of the GI tract
and the nutritional needs of the infant.
(B) Production of milk is supply and demand. A common growth spurt occurs at 3 months of age, and
more frequent nursing will increase the milk supply to satisfy the infant.
(C) Supplementation will decrease the infant's appetite and in turn decrease the milk supply. When the
infant nurses less often or with less vigor, the amount of milk produced decreases.
(D) Rigid feeding schedules lead to a decreased milk supply, whereas frequent nursing signals the mother's
body to produce a correspondingly increased amount of milk.
454. The nurse is teaching a client how to perform monthly testicular self-examination (TSE) and states
that it is best to perform the procedure right after showering. This statement is made by the nurse based on
the knowledge that:
A. The client is more likely to remember to perform the TSE when in the nude
B. When the scrotum is exposed to cool temperatures, the testicles become large and bulky
C. The scrotum will be softer and more relaxed after a warm shower, making the testicles easier to palpate
D. The examination will be less painful at this time
Answer: C
Explanation:
(A) Nudity is not a trigger for reminding males to perform TSE.
(B) Testicles become more firm when exposed to cool temperatures, but not large and bulky.
(C) The testicles will be lower and more easily palpated with warmer temperatures. A protective
mechanism of the body to protect sperm production is for the scrotum to pull closer to the body when
exposed to cooler temperatures.

(D) The examination should not be painful.
455. A young child has been placed in a spica cast. The chief concern of the nurse during the first few
hours is:
A. Prevention of neurovascular complications
B. Prevention of loss of muscle tone
C. Immobilization of the affected limb
D. Using heated fans to dry the cast
Answer: A
Explanation:
(A) Because the extremity may continue to swell and the cast could constrict circulation, the nurse should
elevate the limb and observe for capillary refill, warmth, mobility of toes and circulation.
(B) Although muscle tone may diminish over time in the affected limb, this is not the immediate concern.
(C) The limb has been immobilized already by the cast, and therefore immobilization is not a concern.
(D) Heated fans and dryers are discouraged because the outside cast will dry quickly, yet the area beneath
the cast remains wet and could cause burns.
456. A client is going to have a pneumonectomy in the morning. She had a previous negative surgical
experience, is talking rapidly, and has an increased pulse and respiratory rate. Nursing interventions for
this client should include:
A. Providing opportunities to ask questions and talk about concerns
B. Providing distractors such as reading or watching television
C. Telling her that she should not be so nervous and assuring her that everything will be OK
D. Reminding her that this surgery is not as extensive as her past surgery was
Answer: A
Explanation:
(A) This intervention will help to clarify any misunderstandings about the surgery and give the client an
opportunity to verbalize concerns about the surgery.
(B) Distractors will not alleviate the
preoperative anxiety that the client is experiencing. This response actually denies the client's anxiety.
(C) This intervention is false assurance and denies that anxiety is a normal response to the threat of
surgery.

(D) Psychological responses are not directly related to the extent of the surgery, because they are
influenced by the client's past experiences.
457. A 40-year-old client is admitted to the hospital for tests to diagnose cancer. Since his admission, he
has become dependent and demanding to the nursing staff. The nurse identifies this behavior as which
defense mechanism?
A. Denial
B. Displacement
C. Regression
D. Projection
Answer: C
Explanation:
(A) Denial is the disowning of consciously intolerable thoughts.
(B) Displacement is the referring of a feeling or emotion from one person, object, or idea to another.
(C) Regression is returning to an earlier stage of development.
(D) Projection is attributing one's own thoughts, feelings, or impulses to another person.
458. A mother who is breast-feeding her newborn asks the RN, "How can I express milk from my breasts
manually?" The RN tells her that the correct method for manual milk expression includes using the thumb
and the index finger to:
A. Alternately compress and release each nipple
B. Roll the nipple and gently pull the nipple forward
C. Slide the thumb and index finger forward from the outer border of the areola toward the end of the
nipple
D. Compress and release each breast at the outer border of the areola
Answer: D
Explanation:
(A) Manipulation of nipples will cause soreness and trauma.
(B) Pulling the nipples will cause discomfort and soreness.
(C) Sliding the thumb and index finger forward over the nipple will cause soreness.
(D) The best method to express milk from the breast is to position the thumb and index finger at the outer
border of the areola and compress. This is the location of the milk sinuses.

459. A 37-year-old client has been taking antipsychotic medication for the past 10 days. The nurse
observes her walking with a shuffling gait and postural rigidity and notes a masklike expression on her
face. Which side effect is this client exhibiting?
A. Dystonia
B. Parkinsonism
C. Tardive dyskinesia
D. Akathesia
Answer: B
Explanation:
(A) This answer is incorrect. Dystonia refers to severe, painful muscle contractions.
(B) This answer is correct. Parkinsonism commonly occurs approximately 1-2 weeks after initiation of
antipsychotic drug therapy. Traditional signs are masklike facies, postural rigidity, shuffling gait, and
resting tremor.
(C) This answer is incorrect. Tardive dyskinesia is characterized by involuntary muscle movements of the
face, jaw, and tongue.
(D) This answer is incorrect. Akathesia is motor restlessness.
460. Cheyne-Stokes respiratory pattern can be associated with which of the following conditions?
A. Diabetic ketoacidosis
B. Fever
C. Increased intracranial pressure
D. Spinal meningitis
Answer: C
Explanation:
Explanation
(A) Kussmaul's respirations are associated with diabetic ketoacidosis, severe haemorrhage, peritonitis,
renal failure, and uremia.
(B) Tachypnea (respiratory rate >25 breaths/min) is often associated with fever.
(C) Cheyne-Stokes respiratory pattern is most often associated with increased intracranial pressure
secondary to changes in pressure in the cerebral and cerebellar areas.
(D) Biot's breathing is most frequently associated with spinal meningitis.
461. A nurse should carefully monitor a client for the following side effect of MgSO 4:

A. Visual blurring
B. Tachypnea
C. Epigastric pain
D. Respiratory depression
Answer: D
Explanation:
(A, C) The nurse should provide good distractors because these symptoms indicate that PIH has become
more severe and may precede the convulsive or eclamptic phase.
(B) This is the opposite side effect of this medication.
(D) This is a common side effect of this medication and needs to be monitored and recorded frequently.
462. A client had a cardiac catheterization with angiography and thrombolytic therapy with streptokinase.
The nurse should initiate which of the following interventions immediately after he returns to his room?
A. Place him on NPO restriction for 4 hours.
B. Monitor the catheterization site every 15 minutes.
C. Place him in a high Fowler position.
D. Ambulate him to the bathroom to void.
Answer: B
Explanation:
(A) A contrast dye, iodine, is used in this procedure. This dye is nephrotoxic. The client must be
encouraged to drink plenty of liquids to assist the kidneys in eliminating the dye.
(B) Streptokinase activates plasminogen, dissolving fibrin deposits. To prevent bleeding, pressure is
applied at the insertion site. The client is assessed for both internal and external bleeding.
(C) The extremity used for the insertion site must be kept straight and be immobilized because of the
potential for bleeding.
(D) The client is kept on bed rest for 8-12 hours following the procedure because of the potential for
bleeding.
463. When evaluating a client with symptoms of shock, it is important for the nurse to differentiate
between neurogenic and hypovolemic shock. The symptoms of neurogenic shock differ from hypovolemic
shock in that:
A. In neurogenic shock, the skin is warm and dry
B. In hypovolemic shock, there is a bradycardia

C. In hypovolemic shock, capillary refill is less than 2 seconds
D. In neurogenic shock, there is delayed capillary refill
Answer: A
Explanation:
(A) Neurogenic shock is caused by injury to the cervical region, which leads to loss of sympathetic
control. This loss leads to vasodilation of the vascular beds, bradycardia resulting from the lack of
sympathetic balance to parasympathetic stimuli from the vagus nerve, and the loss of the ability to sweat
below the level of injury. In neurogenic shock, the client is hypotensive but brady cardiac with warm, dry
skin.
(B) In hypovolemic shock, the client is hypotensive and tachycardiac with cool skin.
(C) In hypovolemic shock, the capillary refill would be>5 seconds.
(D) In neurogenic shock, there is no capillary delay, the vascular beds are dilated, and peripheral flow is
good.
464. A child is admitted with severe headache, fever, vomiting, photophobia, drowsiness, and stiff neck
associated with viral meningitis. She will be more comfortable if the nurse:
A. Dims the lights in her room
B. Encourages her to breathe slowly and deeply
C. Offers sips of warm liquids
D. Places a large, soft pillow under her head
Answer: A
Explanation:
(A) The discomfort of photophobia is alleviated by dimming the lights.
(B) Helping the child to breathe slowly and deeply may help to reduce anxiety, but it will not alleviate
other discomforts of viral meningitis.
(C) It is important to maintain fluid balance, but sips of warm liquids do not alleviate the discomforts of
meningitis.
(D) A large, soft pillow under her head causing neck flexion is likely to increase her discomfort owing to
stretching of the meninges.
465. A 54-year-old client is admitted to the hospital with a possible gastric ulcer. He is a heavy smoker.
When discussing his smoking habits with him, the nurse should advise him to:
A. Smoke low-tar, filtered cigarettes

B. Smoke cigars instead
C. Smoke only right after meals
D. Chew gum instead
Answer: C
Explanation:
(A, B, D) Cigarettes, cigars, and chewing gum would stimulate gastric acid secretion.
(C) Smoking on a full stomach minimizes effect of nicotine on gastric acid.
466. With a geriatric client, the nurse should also assess whether he has been obtaining a yearly
vaccination against influenza. Why is this assessment important?
A. Influenza is growing in our society.
B. Older clients generally are sicker than others when stricken with flu.
C. Older clients have less effective immune systems.
D. Older clients have more exposure to the causative agents.
Answer: C
Explanation:
(A) Although influenza is common, the elderly are more at risk because of decreased effectiveness of their
immune system, not because the incidence is increasing.
(B) Older clients have the same degree of illness when stricken as other populations.
(C) As people age, their immune system becomes less effective, increasing their risk for influenza.
(D) Older clients have no more exposure to the causative agents than do school-age children, for example.
467. A 16-year-old client reports a weight loss of 20% of her previous weight. She has a history of food
binges followed by self-induced vomiting (purging). The nurse should suspect a diagnosis of:
A. Anorexia nervosa
B. Anorexia hysteria
C. Bulimia
D. Conversion reaction
Answer: C
Explanation:
(A) Anorexia nervosa is characterized by self-starvation.
(B) Anorexia hysteria is not a known disease or disorder.
(C) Bulimia is characterized by food binges and self-induced vomiting.

(D) Conversion reaction is a defense mechanism.
468. When administering phenytoin (Dilantin) to a child, the nurse should be aware that a toxic effect of
phenytoin therapy is:
A. Stephens-Johnson syndrome
B. Folate deficiency
C. Leukopenic aplastic anemia
D. Granulocytosis and nephrosis
Answer: A
Explanation:
(A) Stephens-Johnson syndrome is a toxic effect of phenytoin.
(B) Folate deficiency is a side effect of phenytoin, but not a toxic effect.
(C) Leukopenic aplastic anemia is a toxic effect of carbamazepine (Tegretol).
(D) Granulocytosis and nephrosis are toxic effects of trimethadione (Tridione).
469. The child with iron poisoning is given IV deforoxamine mesylate (Desferal). Following
administration, the child suffers hypotension, facial flushing, and urticaria. The initial nursing intervention
would be to:
A. Discontinue the IV
B. Stop the medication, and begin a normal saline infusion
C. Take all vital signs, and report to the physician
D. Assess urinary output, and if it is 30 mL an hour, maintain current treatment
Answer: B
Explanation:
(A) The IV line should not be discontinued because other IV medications will be needed.
(B) Stop the medication and begin a normal saline infusion. The child is exhibiting signs of an allergic
reaction and could go into shock if the medication is not stopped. The line should be kept opened for other
medication.
(C) Taking vital signs and reporting to the physician is not an adequate intervention because the IV
medication continues to flow.
(D) Assessing urinary output and, if it is 30 mL an hour, maintaining current treatment is an inappropriate
intervention owing to the child's obvious allergic reaction.

470. A male client has been an insulin-dependent diabetic for approximately 30 years. He frequently
indulges in high sugar foods and forgets to take his insulin. He has not experienced acute diabetic
emergencies over the years but is now beginning to demonstrate symptoms of diabetic peripheral
neuropathy. This distresses him because dancing is one of his favorite pastimes. He decides to question his
wife's home health nurse about diabetic peripheral neuropathy. The nurse points out his noncompliance to
his diabetic diet and insulin regimen. The client answers the nurse, "It has been my experience that the
diabetic diet is very difficult to follow. As far as the insulin, isn't a fellow allowed to forget now and then?"
The client's actions and response best demonstrate:
A. Depression
B. Anger
C. Denial
D. Bargaining
Answer: C
Explanation:
(A) Depression may be an underlying feature, but it is not evident from limited data presented here.
(B) Anger is not exhibited in his response.
(C) Denial is evident in the client's actions; through the years, he has had a casual approach to his illness.
He only becomes concerned when bodily changes affect his present lifestyle, when in fact he should have
been concerned all along. His verbal response also reflects denial.
(D) There is no evidence of bargaining in the client's actions or verbal response.
471. After 3 weeks of treatment, a severely depressed client suddenly begins to feel better and starts
interacting appropriately with other clients and staff. The nurse knows that this client has an increased risk
for:
A. Suicide
B. Exacerbation of depressive symptoms
C. Violence toward others
D. Psychotic behavior
Answer: A
Explanation:
(A) When the severely depressed client suddenly begins to feel better, it often indicates that the client has
made the decision to kill himself or herself and has developed a plan to do so.
(B) Improvement in behavior is not indicative of an exacerbation of depressive symptoms.

(C) The depressed client has a tendency for self-violence, not violence toward others.
(D) Depressive behavior is not always accompanied by psychotic behavior.
472. The most appropriate method of evaluating whether the diet of a child with cystic fibrosis is meeting
his caloric needs is:
A. Careful monitoring of weight loss or gain
B. Carefully recording amounts and types of foods ingested
C. Keeping a strict account of the number of calories ingested
D. Keeping a careful account of the amount of pancreatic enzymes ingested
Answer: A
Explanation:
(A) Consistent weight gain, even if it is slow, is an indication that the child is eating and digesting
sufficient calories.
(B) Recording how much the child eats is useful, but it is not an indicator of how well his body is using the
foods consumed.
(C) Counting calories will indicate how much he is eating, but it will not reflect whether or not the foods
are properly digested.
(D) Keeping track of the enzyme intake will indicate compliance with medication but not whether the
child is getting sufficient calories.
473. A male client seeks counseling after his wife of 19 years threatened to divorce him. For most of their
marriage, he has physically and verbally abused her. When asked about his behavior in the process of the
nursing assessment, the client states, "I was mean to my wife because she insists on cooking meals and
wearing clothes that I do not like." This defense mechanism is an example of:
A. Repression
B. Regression
C. Reaction formation
D. Rationalization
Answer: D
Explanation:
(A) Repression is blocking a desire from conscious expression. The client is conscious of his desires.
(B) Regression is returning to an earlier form of expression, which is not demonstrated here.

(C) Reaction formation is acting out the opposite of true feelings. The client felt anger concerning his
wife's cooking and acted out his feelings.
(D) Rationalization is unconsciously falsifying an experience by giving a "rational" explanation. The client
is attempting to justify his behavior by giving an explanation.
474. Which of the following medications requires close observation for bronchospasm in the client with
chronic obstructive pulmonary disease or asthma?
A. Verapamil (Isoptin)
B. Amrinone (Inocor)
C. Epinephrine (Adrenalin)
D. Propranolol (Inderal)
Answer: D
Explanation:
(A) Verapamil has the respiratory side effect of nasal or chest congestion, dyspnea, shortness of breath
(SOB), and wheezing.
(B) Amrinone has the effect of increased contractility and dilation of the vascular smooth muscle. It has no
noted respiratory side effects.
(C) Epinephrine has the effect of bronchodilation through stimulation.
(D) Propranolol, esmolol, and labetalol are all - blocking agents, which can increase airway resistance and
cause bronchospasms.
475. Signs and symptoms of an allergy attack include which of the following?
A. Wheezing on inspiration
B. Increased respiratory rate
C. Circumoral cyanosis
D. Prolonged expiration
Answer: D
Explanation:
(A) Wheezing occurs during expiration when air movement is impaired because of constricted edematous
bronchial lumina.
(B) Respirations are difficult, but the rate is frequently normal.
(C) The circumoral area is usually pale. Cyanosis is not an early sign of hypoxia.
(D) Expiration is prolonged because the alveoli are greatly distended and air trapping occurs.

476. A 4-week-old infant is admitted to the emergency room in respiratory distress. Which of the following
statements indicates the nurse's knowledge of the anatomy of the respiratory system in paediatric clients?
A. The diameter of the trachea is much smaller in children than in adults.
B. The tongue is proportionally smaller in children than in adults.
C. The paediatric airway is more rigid than that of the adults.
D. The length of the paediatric airway is longer in children than in adults.
Answer: A
Explanation:
(A) The airway in children is much smaller than it is in adults. The diameter of the trachea in the newborn
is 4 mm and that of the adult is 20 mm. A small change in the diameter of the airway can make a major
difference in the paediatric client.
(B) The tongue is proportionally larger in children and fills most of the oral cavity, thereby decreasing air
space.
(C) The entire paediatric airway is elastic. Elasticity diminishes with age, however.
(D) The distances between respiratory structures are shorter than that of adults, and therefore organisms
are able to move more rapidly down the throat, leading to more extensive respiratory involvement.
477. While the RN is assessing a mother's perineum on her 2nd postpartum day after having a vaginal
delivery, the RN notes a large ecchymotic area located to the left of the mother's perineum.
Which one of the following interventions should the RN initiate at this time?
A. Have the client expose the area to air.
B. Apply ice to the perineum.
C. Encourage the client to take warm sitz baths.
D. Inform the physician.
Answer: C
Explanation:
(A) The area is bruised and painful. This action would do nothing to help with the healing process of the
perineum or to provide comfort.
(B) Ice is effective immediately after birth to reduce edema and discomfort, but not on the 2nd postpartum
day.
(C) Sitz baths are useful if the perineum has been traumatized, because the moist heat increases circulation
to the area to promote healing, relaxes tissue, and decreases edema.

(D) The physician is not notified of bruising, but if a hematoma is present, then the physician is notified.
478. A 48-year-old client is being seen in her physician's office for complaints of indigestion, heartburn,
right upper quadrant pain, and nausea of 4 days' duration, especially after meals. The nurse realizes that
these symptoms may be associated with cholecystitis and therefore would check for which specific sign
during the abdominal assessment?
A. Cullen's sign
B. Rebound tenderness
C. Murphy's sign
D. Turner's sign
Answer: C
Explanation:
(A) This sign is a faint blue discoloration around the umbilicus found in clients who have haemorrhagic
pancreatitis.
(B) This sign indicates areas of inflammation within the peritoneum, such as with appendicitis. It is a deep
palpation technique used on a nontender area of the abdomen, and when the palpating hand is removed
suddenly, the client experiences a sharp, stabbing pain at an area of peritoneal inflammation.
(C) This sign is considered positive with acute cholecystitis when the client is unable to take a deep breath
while the right upper quadrant is being deeply palpated. The client will elicit a sudden, sharp gasp, which
means the gallbladder is acutely inflamed.
(D) This is a sign of acute haemorrhagic pancreatitis and manifests as a green or purple discoloration in the
flanks.
479. The nurse is caring for a laboring client. Assessment data include cervical dilation 9 cm; contractions
every 1-2 minutes; strong, large amount of "bloody show." The most appropriate nursing goal for this
client would be:
A. Maintain client's privacy.
B. Assist with assessment procedures.
C. Provide strategies to maintain client control.
D. Enlist additional caregiver support to ensure client's safety.
Answer: C
Explanation:
(A) Privacy may help the laboring client feel safer, but measures that enhance coping take priority.

(B) The frequency of assessments do increase in transition, but helping the client to maintain control and
cope with this phase of labor takes on importance.
(C) This laboring client is in transition, the most difficult part of the first stage of labor because of
decreased frequency, increased duration and intensity, and decreased resting phase of the uterine
contraction. The client's ability to cope is most threatened during this phase of labor, and nursing actions
are directed toward helping the client to maintain control.
(D) Safety is a concern throughout labor, but helping the client to cope takes on importance in transition.
480. The most important reason to closely assess circumferential burns at least every hour is that they may
result in:
A. Hypovolemia
B. Renal damage
C. Ventricular arrhythmias
D. Loss of peripheral pulses
Answer: D
Explanation:
(A) Hypovolemia could be a result of fluid loss from thermal injury, but not as a result of the
circumferential injury.
(B) Renal damage is typically seen because of prolonged hypovolemia or myoglobinuria.
(C) Electrical injuries and electrolyte changes typically cause arrhythmias in the burn client.
(D) Full-thickness circumferential burns are nonelastic and result in an internal tourniquet effect that
compromises distal blood flow when the area involved is an extremity. Circumferential full thickness torso
burns compromise respiratory motion and, when extreme, cardiac return.
481. A 30-year-old client is exhibiting auditory hallucinations. In working with this client, the nurse would
be most effective if the nurse:
A. Encourages the client to discuss the voices
B. Attempts to direct the client's attention to the here and now
C. Exhibits sincere interest in the delusional voices
D. Gives the medication as necessary for the acting-out behavior
Answer: B
Explanation:
(A) This answer is incorrect. Encouraging discussion of the voices will reinforce the delusion.

(B) This answer is correct. The nurse should appropriately present reality.
(C) This answer is incorrect. Showing interest would reinforce the delusional system.
(D) This answer is incorrect. The statement only indicates that the client is hearing voices. It does not state
that the client is acting out.
482. A client is hyperactive and not sleeping. She will not remain at the table during mealtime. She is
getting very limited calories and is using a lot of energy in her hyperactive state. The most therapeutic
nursing action is to:
A. Insist that she remain at the table and eat a balanced diet.
B. Order a high-calorie diet with supplements.
C. Provide nutritious finger foods several times a day.
D. Offer to go to the dining room with her and allow her to open the food and inspect what she eats.
Answer: C
Explanation:
(A) The client is not able to sit for long periods. Forcing her to remain at the table will increase her anxiety
and cause her to become hostile.
(B) This action will not ensure that the client eats what is ordered. Dietary orders are not within the nurse's
scope of practice.
(C) Providing finger foods increases the likelihood of eating for hyperactive persons. They may be eating
"on the run."
(D) These clients are not suspicious of the food or insecure in moving about the unit alone.
483. Which of the following would have the physiological effect of decreasing intracranial pressure (ICP)?
A. Increased core body temperature
B. Decreased serum osmolality
C. Administration of hypo-osmolar fluids
D. Decreased PaCO2
Answer: D
Explanation:
(A) An increase in core body temperature increases metabolism and results in an increase in ICP.
(B) Decreased serum osmolality indicates a fluid overload and may result in an increase in ICP.

(C) Hypoosmolar fluids are generally voided in the neurologically compromised. Using IV fluids such as
D5W results in the dextrose being metabolized, releasing free water that is absorbed by the brain cells,
leading to cerebral edema.
(D) Hypercapnia and hypoventilation, which cause retention of CO2 and lead to respiratory acidosis, both
increase ICP. CO2 is the most potent vasodilator known.
484. An 8-week-old infant has been diagnosed with gastroesophageal reflux. The nurse is teaching the
infant's mother to care for the infant at home. Which one of the following statements by the nurse is
appropriate regarding the infant's home care?
A. "Lay the infant flat on her left side after feeding."
B. "Feed the infant every 4 hours with half-strength formula."
C. "Antacids need to be given an hour before feeding."
D. "Play activities should be carried out before instead of after feedings."
Answer: D
Explanation:
(A) Elevating the child's head to a 30-degree angle is the recommended position for gastroesophageal
reflux. The supine position predisposes the child to aspiration.
(B) Small, frequent feedings with thickened formula are recommended to minimize vomiting.
(C) Antacids should be given at the same time as the feeding to improve their buffering action.
(D) The infant should be kept still after feedings to reduce the risk of vomiting and aspiration. Vigorous
activities should be carried out before feedings.
485. Assessment of a newborn for Apgar scoring includes observation for:
A. Pupil response
B. Respiratory rate
C. Heart rate
D. Babinski's reflex
Answer: C
Explanation:
(A) Pupil response should be assessed but is not part of Apgar scoring.
(B) Respiratory effort is an essential part of Apgar scoring, not respiratory rate.
(C) Heart rate is the most critical component of Apgar scoring.
(D) Assessment of Babinski's reflex is not a component of Apgar scoring.

486. A client presented herself to the mental health center, describing the following symptoms: a weight
loss of 20 lb in the past 2 months, difficulty concentrating, repeated absences from work due to "fatigue,"
and not wanting to get dressed in the morning. She leaves her recorded message on her telephone and has
lost interest in answering the phone or doorbell. The nurse's assessment of her behavior would most likely
be:
A. Deep depression
B. Psychotic depression
C. Severe anxiety
D. Severe depression
Answer: D
Explanation:
(A) A client in deep depression would have been brought to the mental health center and would not be
physically able to seek help for herself.
(B) She is not manifesting psychotic symptoms in her behaviors.
(C) The client's symptoms are more indicative of depression than anxiety.
(D) Although the client was able to bring herself to the mental health center, the extent of her weight loss
and the interference of symptoms with activities of daily living indicate that she is severely depressed.
487. A 58-year-old client on a general surgery unit is scheduled for transurethral resection of the prostate
(TURP) in 2 hours. The nurse explains to the client that this procedure means:
A. Removal of the prostate tissue by way of a lower abdominal midline incision through the bladder and
into the prostate gland
B. Removal of prostate tissue by a resectoscope that is inserted through the penile urethra
C. Removal of the prostate tissue by an open surgical approach through an incision between the ischial
tuberosities, the scrotum, and the rectum
D. Removal of prostate tissue by an open surgical approach through a low horizontal incision, bypassing
the bladder, to the prostate gland
Answer: B
Explanation:
(A) This describes a suprapubic (trans-vesical) prostatectomy procedure.
(B) This is the correct description of a TURP procedure.
(C) This describes a perineal prostatectomy procedure.

(D) This describes a retropubic (extravesical) prostatectomy procedure.
488. Parents of a child with rheumatic fever express concern that she will always be arthritic. The nurse
discusses their concerns and tells them the joint pain usually:
A. Subsides in <3 weeks
B. Is relieved by aspirin
C. Is responsive to ibuprofen (Motrin)
D. Subsides in 3-6 days
Answer: A
Explanation:
(A) Joints usually remain inflamed and tender until the disease runs its course in<3 weeks.
(B) This response does not answer the question of whether she will always be arthritic.
(C) This response does not answer the question asked.
(D) The disease takes500 mL
should be reported to the physician, because an occlusion of some type, caused by a retained gallstone or
an inflammatory process within the biliary drainage system, is evident.
543. A 71-year-old client fell and injured her left leg while cooking in the kitchen. Her husband calls the
ambulance, and she is taken to the emergency department at a local hospital. X-ray reports confirm that
she has an intertrochanteric fracture of the left femur. Her left leg will require skeletal traction initially and
then surgery. The nurse knows that this type of traction will be used:
A. By inserting pins to provide steady pull on the bone
B. To suspend the leg in a sling without pull on the extremity
C. Intermittently to place a pull over the pelvis and lower spine
D. With weights at both ends of the bed to maintain pull on the upper extremity
Answer: A
Explanation:
(A) Skeletal traction is the application of traction directly to bone with the use of pins and wires or tongs
for the purpose of providing a strong, steady, continuous longitudinal pull on the bone. It is indicated for
preoperative immobilization and positioning of hip and femur fractures.

(B) A type of skeletal traction (balanced suspension with a Thomas splint and Pearson attachment) uses a
sling to support the extremity, but it also uses weights to provide a strong, steady continuous pull on the
extremity. A sling is used instead of pins.
(C) Pelvic traction provides an intermittent pull over the pelvis and bone, whereas skeletal traction is
continuous. Pelvic traction does not use pins.
(D) Skeletal traction uses weights at the end of the bed to provide a continuous pull on long bones.
Weights are not applied to both ends of the bed.
544. Dietary planning is an essential part of the diabetic client's regimen. The American
Diabetes Association recommends which of the following caloric guidelines for daily meal planning?
A. 50% complex carbohydrate, 20%-25% protein, 20%-25% fat
B. 45% complex carbohydrate, 25%-30% protein, 30%-35% fat
C. 70% complex carbohydrate, 20%-30% protein, 10%-20% fat
D. 60% complex carbohydrate, 12%-15% protein, 20%-25% fat
Answer: D
Explanation:
(A) The percentage of carbohydrates is too low to maintain blood sugar levels. The percent range of
protein is too high and may cause extra workload on the kidney as it is metabolized.
(B) The percentage of carbohydrates is too low to maintain blood sugar levels. The percent range of
protein is too high and may cause extra workload on the kidney.
(C) The percentage of carbohydrates is too high; the percent range of protein is too high, and of fat, too
low.
(D) This combination provides enough carbohydrates to maintain blood glucose levels, enough protein to
maintain body repair, and enough fat to ensure palatability.
545. A mother came to the paediatric clinic with her 17- month-old child. The mother would like to begin
toilet training. What should the nurse teach her about implementing toilet training?
A. Take two or three favorite toys with the child.
B. Have a child-sized toilet seat or training potty on hand.
C. Explain to the child she is going to "void" and "defecate."
D. Show disapproval if she does not void or defecate.
Answer: B
Explanation:

(A) Giving her toys will distract her and interfere with toilet training because of inappropriate
reinforcement.
(B) A child-sized toilet seat or training potty gives a child a feeling of security.
(C) She should use words that are age appropriate for the child.
(D) Children should be praised for cooperative behavior and/or successful evacuation.
546. A 72-year-old client with a new colostomy is being evaluated at the clinic today for constipation.
When discussing diet with the client, the nurse recognizes that which one of the following foods most
likely caused this problem?
A. Fried chicken
B. Eggs
C. Tapioca
D. Cabbage
Answer: C
Explanation:
(A) Fried, greasy food, such as fried chicken, will produce diarrhoea like stools in individuals with all
types of GI ostomies.
(B) Eggs will cause odor-producing stools in individuals with all types of GI ostomies.
(C) Tapioca and rice products will cause constipation in individuals with all types of GI ostomies.
(D) Cabbage will cause odor-producing and flatus-producing stools in individuals with all types of GI
ostomies.
547. A client hospitalized with a medical diagnosis of adjustment disorder versus personality disorder
states, "Nobody cares about the clients." The nurse's most effective response would be:
A. "How can you say that I don't care? We just met."
B. "What makes you think the nurses don't care?"
C. "You will feel differently about us in a few days."
D. "You seem angry. Tell me more about how you feel."
Answer: D
Explanation:
(A) This statement is a defensive response that places the nurse in a vulnerable countertransference
position, and at the same time, fails to challenge the client's "splitting" behavior.

(B) This statement is a defensive response by the nurse. In addition, this type of nontherapeutic statement
requests that the client explain the reasons for her behavior, a difficult task for an individual with limited
insight.
(C) This statement is a nontherapeutic response that both ignores the intensity of the client's emotions and
the dynamics underlying "splitting" behavior.
(D) By simultaneously acknowledging the client's emotional intensity and gently challenging her
"splitting" behavior, the nurse addresses the client's current distortions and prepares for further
interventions with angry or ambivalent feelings.
548. A 15-year-old client was diagnosed as having cystic fibrosis at 8 months of age. He is in the hospital
for a course of IV antibiotic therapy and vigorous chest physiotherapy. He has a poor appetite. The nurse
can best help him to meet the desired outcome of consuming a prescribed number of calories by:
A. Including the client in planning sessions to select the type of meal plan and foods for his diet
B. Working with the nutritionist to devise a diet with significantly increased calories
C. Selecting foods for the client's diet that are high in calories and instituting a strict calorie count
D. Constantly providing him with chips, dips, and candies, because the number of calories consumed is
more important than the quality of foods
Answer: A
Explanation:
(A) The adolescent knows what he likes and will be more likely to eat if he has some control over his diet.
(B) The nurses and nutritionist can plan an excellent diet, but it will not help the adolescent unless he eats
it.
(C) Eating is already a chore for this client. Adding a strict calorie count could make it even more
burdensome.
(D) Fats are particularly difficult for the cystic fibrosis client to digest. He does need a healthful diet, not
just more calories.
549. Nursing assessment of early evidence of septic shock in children at risk includes:
A. Fever, tachycardia, and tachypnea
B. Respiratory distress, cold skin, and pale extremities
C. Elevated blood pressure, hyperventilation, and thready pulses
D. Normal pulses, hypotension, and oliguria
Answer: A

Explanation:
(A) Fever, tachycardia, and tachypnea are the classic early signs of septic shock in children.
(B) Respiratory distress, cold skin, and pale extremities are later signs of septic shock.
(C) Elevated blood pressure, hyperventilation, and thready pulses are later signs of septic shock.
(D) Normal pulses, hypotension, and oliguria are not early signs of septic shock.
550. A 2-year-old boy is in the hospital outpatient department for observation after falling out of his crib
and hitting his head. The nurse calls the physician to report:
A. Evidence of perineal irritation
B. Pulse fell from 102 to 96
C. Pulse increased from 96 to 102
D. Temperature rose to 102_F rectally
Answer: D
Explanation:
(A) Perineal irritation needs to be addressed, but it is probably not necessary to call the physician.
(B) This fall in pulse rate remains within normal limits and is probably insignificant. It is important to
monitor for continued change.
(C) This rise in pulse rate is probably not significant, but it is important to monitor for continued change.
(D) This temperature is above normal limits and needs medical investigation. It may or may not be related
to the head injury.
551. A 7-year-old child is brought to the ER at midnight by his mother after symptoms appeared abruptly.
The nurse's initial assessment reveals a temperature of 104.5F (40.3C), difficulty swallowing, drooling,
absence of a spontaneous cough, and agitation. These symptoms are indicative of which one of the
following?
A. Acute tracheitis
B. Acute spasmodic croup
C. Acute epiglottis
D. Acute laryngotracheobronchitis
Answer: C
Explanation:
(A) Clinical manifestations of acute tracheitis include a 2-3 day history of URI, croupy cough, stridor,
purulent secretions, high fever.

(B) Clinical manifestations of spasmodic croup include a history of URI, croupy cough, stridor, dyspnea,
low-grade fever, and a slow progression. The age group most affected is 3 months to 3 years.
(C) Three clinical observations have been found to be predictive of epiglottitis: the presence of drooling,
absence of spontaneous cough, and agitation. Epiglottitis has a rapid onset that is accompanied by high
fever and dysphagia.
(D) Clinical manifestations of acute laryngotracheobronchitis (LTB) include slow onset with a history of
URI, low-grade fever, stridor, brassy cough, and irritability.
552. At 38 weeks' gestation, a client is in active labor. She is using her Lamaze breathing techniques. The
RN is coaching her breathing and encouraging her to relax and work with her contractions. Which one of
the following complaints by the client will alert the RN that she is beginning to hyperventilate with her
breathing?
A. "I am cold."
B. "I have a backache."
C. "I feel dizzy."
D. "I am nauseous."
Answer: C
Explanation:
(A) Cold is not a symptom of hyperventilation. This could be due to the temperature of the room.
(B) Backache is not a symptom of hyperventilation. This is probably due to the gravid uterus and its effect
on the back muscles, or it may be due to the client's position in bed.
(C) Dizziness is the first symptom of hyperventilation. It occurs because the body is eliminating too much
CO2.
(D) Nausea is not a symptom of hyperventilation. It could be a symptom of pain.
553. At 16 weeks' gestation, a pregnant client is admitted to the maternity unit to have a McDonald
procedure (cerclage) done. She tells the RN who is admitting her to the unit that her physician had
explained what this procedure was, but that she did not understand. The RN explains to the client that the
purpose for this procedure is to:
A. Reinforce an incompetent cervix
B. Repair the amniotic sac
C. Evaluate cephalopelvic disproportion
D. Dilate the cervix

Answer: A
Explanation:
(A) The treatment most commonly uses the Shirodkar-Barter procedure (McDonald procedure) or cerclage
to enforce the weakened cervix by encircling it with a suture at the level of the internal os.
(B) There is no known procedure that is used to repair the amniotic sac.
(C) Cephalopelvic disproportion is evaluated later in pregnancy. It is not related to this procedure.
(D) No procedure is done to dilate the cervix at 16 weeks' gestation unless the pregnancy is to be
terminated.
554. While the nurse is taking a male client's blood pressure, he makes flirtatious remarks to her. The nurse
will handle this effectively if she:
A. Politely tells the client, "Keep your hands off "
B. Ignores the remarks and hopes he will not try it again
C. Confronts the remarks but attempts not to reject the client
D. Leaves the room in order to compose herself
Answer: C
Explanation:
(A) This response does not recognize normal feelings of attraction and rejects the client.
(B) By ignoring the situation, the nurse has not set limits to discourage other remarks or perhaps more
sexually aggressive behavior.
(C) By confronting the remarks, she can recognize that his feelings of attraction may be normal but are not
appropriate within the context of their nurse-client relationship.
(D) Leaving the room does not deal with setting limits for future interactions.
555. A 74-year-old obese man who has undergone open reduction and internal fixation of the right hip is 8
days postoperative. He has a history of arthritis and atrial fibrillation. He admits to right lower leg pain,
described as "a cramp in my leg." An appropriate nursing action is to:
A. Assess for pain with plan tiflexion
B. Assess for edema and heat of the right leg
C. Instruct him to rub the cramp out of his leg
D. Elevate right lower extremity with pillows propped under the knee
Answer: B
Explanation:

(A) Calf pain with dorsiflexion of the foot (Homans' sign) can be a sign of a deep venous thrombosis;
however, it is not diagnostic of the condition.
(B) Swelling and warmth along the affected vein are commonly observed clinical manifestations of a deep
venous thrombosis as a result of inflammation of the vessel wall.
(C) Rubbing or massaging of the affected leg is contraindicated because of the risk of the clot breaking
loose and becoming an embolus.
(D) A pillow behind the knee can be constricting and further impair blood flow.
556. During his hospitalization, a 3-year-old child has become unusually aggressive in his play activities.
His parents report this change in behavior to the primary nurse. How could the nurse explain the child's
change in behavior?
A. Deep-seated feelings of hostility
B. A lack of interest in socializing
C. Usual behavior for this child
D. A coping response
Answer: D
Explanation:
(A) Unusually aggressive behavior does not indicate a deepseated problem.
(B) A lack of social interest results in poor participation in play activities with peers. Aggression would not
be an expected behavior.
(C) The aggressive behavior was newly developed and not a routine behavior.
(D) Play provides the child with opportunities for coping and adaptation. Aggression during the play
activities would indicate a coping response.
557. The nurse knows that children are more susceptible to respiratory tract infections owing to
physiological differences. These childhood differences, when compared to an adult, include:
A. Fewer alveoli, slower respiratory rate
B. Diaphragmatic breathing, larger volume of air
C. Larger number of alveoli, diaphragmatic breathing
D. Rounded shape of chest, smaller volume of air
Answer: D
Explanation:
(A) Although a child has fewer alveoli than an adult, the child's respiratory rate is faster.

(B) Although a child may use diaphragmatic breathing, the adult exchanges a larger volume of air.
(C) The adult has a larger number of alveoli than a child.
(D) The child's chest is rounded whereas the adult chest is more of an oval shape, and the child does
exchange a smaller volume of air than an adult.
558. A parent told the public health nurse that her 6-year-old son has been taking tetracycline for a chronic
skin condition. The parent asked if this could cause any problems for the child. What should the nurse
explain to the parent?
A. Giving tetracycline to a child younger than 8 years may cause permanent staining of his teeth.
B. If you give tetracycline with milk, it may be absorbed readily.
C. The medication should be given to adults, not children.
D. Secondary infections of chronic skin disorders do not respond to antibiotics.
Answer: A
Explanation:
(A) Tetracycline should be avoided during tooth development because it interferes with enamel formation
and dental pigmentation.
(B) Milk interferes with the absorption of tetracyclines.
(C) Children older than 9 years or past the tooth development stage may be given tetracycline.
(D) Secondary infections of chronic skin disorders may respond to antibiotics such as penicillin or
tetracyclines.
559. A client has begun to exhibit signs of alcohol withdrawal. Her blood pressure has risen from 120/60 to
190/100, pulse is increased from 88 to 110 bpm, and she is irritable and agitated and has gross motor
tremors of the hands. The nurse notifies the doctor. The nurse can anticipate that the doctor will order
which of the following?
A. An opiate such as propoxyphene napsylate (Darvocet)
B. A benzodiazepine such as chlordiazepoxide (Librium)
C. A tricyclic antidepressant such as amitriptyline (Elavil)
D. A phenothiazine such as chlorpromazine (Thorazine)
Answer: B
Explanation:
(A) This answer is incorrect. Benzodiazepines are drugs of choice for alcohol withdrawal.
(B) This answer is correct. The drug has a sedative effect, is safe, and has an anticonvulsant effect.

(C) This answer is incorrect. Amitriptyline is an antidepressant.
(D) This answer is incorrect. Chlorpromazine is most effective in psychotic disorders.
560. A 4-year-old child is being discharged from the hospital after being treated for severe croup. Which
one of the following instructions should the nurse give to the child's mother for the home treatment of
croup?
A. Take him in the bathroom, turn on the hot water, and close the door.
B. Give him a dose of antihistamine.
C. Give large amounts of clear liquids if drooling occurs.
D. Place him near a cool mist vaporizer and encourage crying.
Answer: A
Explanation:
(A) Initial home treatment of croup includes placing the child in an environment of high humidity to
liquefy and mobilize secretions.
(B) Antihistamines should be avoided because they can cause thickening of secretions.
(C) Drooling is a characteristic sign of airway obstruction and the child should be taken directly to the
emergency room.
(D) Crying increases respiratory distress and hypoxia in the child with croup. The nurse should promote
methods that will calm the child.
561. The nurse discovers that a 78-year-old client who received hydralazine (Apresoline) 20 mg 45 minutes
ago has a blood pressure of 70/40 mm Hg. The client has been on this dose of the medication for 3 years.
Which of the following data is most likely significant in relation to the cause of the low blood pressure?
A. Pedal pulses 11 (weak)
B. Twenty-four-hour intake 1000 mL/day for past 2 days
C. Serum potassium 3.3
D. Pulse rate 150 bpm
Answer: B
Explanation:
(A, D) Decreased pulse volume and increased pulse rate are signs of an acute hypotensive episode.
(B) Inadequate fluid volume when taking vasodilators can result in a drop in blood pressure when
vasodilation starts to physiologically occur as an action of the drug.
(C) A potassium level of 3.3 would not be associated with a significant drop in blood pressure.

562. For the past several months, an elderly female client with Alzheimer's disease has experienced
paranoia; hallucinations; and aggressive, disruptive behavior. The family is utilizing haloperidol as needed
to control her behavior. On nursing assessment, you note that the client demonstrates involuntary
movements of the tongue and fingers. This may most likely indicate:
A. Tardive dyskinesia, which may be a side effect of antipsychotic medication
B. Early symptoms of Parkinson's disease
C. A more advanced stage of Alzheimer's disease than previously experienced by the client
D. The need to change her medication from haloperidol to another antipsychotic drug to lessen
symptoms
Answer: A
Explanation:
(A) Tardive dyskinesia is a common side effect of antipsychotic medications such as haloperidol.
Discontinuing the medication can alleviate symptoms.
(B) Although mild tremors are an early sign of Parkinson's disease, haloperidol must be discontinued first
and the client further evaluated.
(C) These symptoms do not necessarily indicate a more advanced stage of Alzheimer's disease.
(D) Most antipsychotic drugs are chemically similar and will produce the same side effects.
563. The following nursing diagnosis is written for a comatose client with cirrhosis of the liver and
secondary splenomegaly-High risk for injury: Increased susceptibility to bleeding related to:
A. Increased absorption of vitamin K
B. Thrombocytopenia due to hypersplenism
C. Diminished function of the Kupffer cells
D. Increased synthesis of the clotting factors
Answer: B
Explanation:
(A) There is a decreased absorption of vitamin K with cirrhosis of the liver. This decrease impairs blood
coagulation and the formation of prothrombin.
(B) Thrombocytopenia, an increased destruction of platelets, occurs secondary to hypersplenism.
(C) A diminished function of the Kupffer cells occurs with cirrhosis of the liver, causing the client to
become more susceptible to infections.

(D) A decrease in the synthesis of fibrinogen and clotting factors VII, IX, and X occurs with cirrhosis of
the liver and increases the susceptibility to bleeding.
564. Which of the following blood gas parameters primarily reflects respiratory function?
A. PCO2
B. CO2 content of the blood
C. HCO3
D. Base excess
Answer: A
Explanation:
(A) The lungs are responsible for regulation of CO2, and this parameter primarily reflects respiratory
function.
(B) CO2 content of the blood is an indirect measure of respiratory function.
(C) HCO3 is a measure of kidney function only and is important in acid-base balance.
(D) Base excess represents the excess of HCO3 and is not reflective of respiratory function.
565. A primigravida is at term. The nurse can recognize the second stage of labor by the client's desire to:
A. Push during contractions
B. Hyperventilate during contractions
C. Walk between contractions
D. Relax during contractions
Answer: A
Explanation:
(A) The second stage of labor is characterized by uterine contractions, which cause the client to bear down.
(B) Slow, deep, rhythmic breathing facilitates the laboring process. Hyperventilation is abnormal breathing
resulting from loss of pain control.
(C) The client should remain on bed rest during labor.
(D) Contractions result in discomfort.
566. A gravida 2 para 1 client delivered a full-term newborn 12 hours ago. The nurse finds her uterus to be
boggy, high, and deviated to the right. The most appropriate nursing action is to:
A. Notify the physician
B. Place the client on a pad count

C. Massage the uterus and re-evaluate in 30 minutes
D. Have the client void and then re-evaluate the fundus
Answer: D
Explanation:
(A) The nurse should initiate actions to remove the most frequent cause of uterine displacement, which
involves emptying the bladder. Notifying the physician is an inappropriate nursing action.
(B) The pad count gives an estimate of blood loss, which is likely to increase with a boggy uterus; but this
action does not remove the most frequent cause of uterine displacement, which is a full bladder.
(C) Massage may firm the uterus temporarily, but if a full bladder is not emptied, the uterus will remain
displaced and is likely to relax again.
(D) The most common cause of uterine displacement is a full bladder.
567. A 25-year-old lawyer who is married with three young children works long hours in an effort to
become a partner in the law firm. Following a recent hospitalization for a bleeding ulcer, he was referred
for therapy to treat this psychophysiological disorder. On meeting with the therapist, he informed him or
her that he was a busy man and did not have much time for this "psych stuff." When guiding the client to
ventilate his feelings, the therapist can expect him to express feelings of:
A. Guilt
B. Shame
C. Despair
D. Anger
Answer: D
Explanation:
(A) Guilt relates to depression.
(B) Shame is not associated with psychophysiological disorders.
(C) Despair relates to depression.
(D) Repressed anger is associated with psychophysiological disorders.
568. The client will be more comfortable and the results more accurate when the nurse prepares the client
for Leopold's maneuvers by having her:
A. Empty her bladder
B. Lie on her left side
C. Place her arms over her head

D. Force fluids 1 hour prior to procedure
Answer: A
Explanation:
(A) A full bladder would cause discomfort and possible urinary incontinence during the exam.
(B) The left side-lying position would not accommodate the exam. The head of the exam table or bed can
be slightly elevated to prevent supine hypotension.
(C) Arms extended over the head would cause the abdomen to be tighter and less easily palpable.
(D) Forcing fluids would encourage a full bladder, which is not desired for the exam.
569. A male client is scheduled for a liver biopsy. In preparing him for this test, the nurse should:
A. Explain that he will be kept NPO for 24 hours before the exam
B. Practice with him so he will be able to hold his breath for 1 minute
C. Explain that he will be receiving a laxative to prevent a distended bowel from applying pressure on the
liver
D. Explain that his vital signs will be checked frequently after the test
Answer: D
Explanation:
(A) There is no NPO restriction prior to a liver biopsy.
(B) The client would need to hold his breath for 5-10 seconds.
(C) There is no pretest laxative given.
(D) Following the test, the client is watched for haemorrhage and shock.
570. A female client at 30 weeks' gestation is brought into the emergency department after falling down a
flight of stairs. On examination, the physician notes a rigid, board like abdomen; FHR in the 160 s; and
stable vital signs. Considering possible abdominal trauma, which obstetric emergency must be anticipated?
A. Abruptio placentae
B. Ectopic pregnancy
C. Massive uterine rupture
D. Placenta previa
Answer: A
Explanation:

(A) Abruptio placentae, the complete or partial separation of the placenta from the uterine wall, can be
caused by external trauma. When haemorrhage is concealed, one sign is a rapid increase in uterine size
with rigidity.
(B) Ectopic pregnancy occurs when the embryo implants itself outside the uterine cavity.
(C) Massive uterine rupture occurs during labor when the uterine contents are extruded through the uterine
wall. It is usually due to weakness from a pre-existing uterine scar and trauma from instruments or an
obstetrical intervention.
(D) Placenta previa is the condition in which the placenta is implanted in the lower uterine segment and
either completely or partially covers the cervical os.
571. When assessing a female child for Turner's syndrome, the nurse observes for which of the following
symptoms?
A. Tall stature
B. Amenorrhea
C. Secondary sex characteristics
D. Gynecomastia
Answer: B
Explanation:
(A) This syndrome is caused by absence of one of the X chromosomes. These children are short in stature.
(B) Amenorrhea is a symptom of Turner's syndrome, which appears at puberty.
(C) Sexual infantilism is characteristic of this syndrome.
(D) Gynecomastia is a symptom in Klinefelter's syndrome.
572. A 27-year-old male client is admitted to the acute care mental health unit for observation. He has
recently lost his job, and his wife told him yesterday that she wants a divorce. The client is placed on
suicide precautions. In assessing suicide potential, the nurse should pay close attention to the client's:
A. Level of insight
B. Thought processes
C. Mood and affect
D. Abstracting abilities
Answer: C
Explanation:

(A) Assessing the client's level of insight is an important part of the mental status exam (MSE), but it does
not reflect suicide potential.
(B) Assessing the client's thought processes is an important part of the MSE, but it does not reflect suicide
potential.
(C) Assessing the client's mood and affect is an important part of the MSE, and it can be a very valuable
indicator of suicide potential. Frequently a client who has decided to proceed with suicide plans will
exhibit a suddenly improved mood and affect.
(D) Assessing a client's abstracting abilities is an important part of the MSE, but it does not reflect suicide
potential.
573. A client has been admitted to the nursing unit with the diagnosis of severe anemia. She is slightly
short of breath, has episodes of dizziness, and complains her heart sometimes feels like it will "beat out of
her chest." The physician has ordered her to receive 2 U of packed red blood cells. The most important
nursing action to be taken is:
A. Starting an 18-gauge IV infusion
B. Having the consent form on the chart
C. Administering the correct blood product to the correct client
D. Transfusing the blood in a 2-hour time frame
Answer: C
Explanation:
(A) An 18-gauge IV is an appropriate size for administering blood; however, client safety demands that the
right blood product must be administered.
(B) The consent form is legally necessary to be on the chart, but client safety is maintained by giving the
correct blood component to the correct client.
(C) Administering the correct blood product to the correct client will maintain physiological safety and
minimize transfusion reactions.
(D) The blood administration should take place over the ordered time frame designated by the physician.
574. A 19-year-old client fell off a ladder approximately 3 ft to the ground. He did not lose consciousness
but was taken to the emergency department by a friend to have a scalp laceration sutured. The nurse
instructs the client to:
A. Clean the sutured laceration twice a day with povidone- iodine (Betadine)
B. Remove his scalp sutures after 5 days

C. Return to the hospital immediately if he develops confusion, nausea, or vomiting
D. Take meperidine 50 mg po q4-6h prn for headache
Answer: C
Explanation:
(A) Povidone-iodine is very irritating to skin and should not be routinely used.
(B) Sutures should not be removed by the client.
(C) Confusion, nausea, vomiting, and behavioral changes may indicate increasing intracranial pressure as a
result of intracerebral bleeding.
(D) Use of a narcotic opiate such as meperidine is not recommended in clients with a possible head injury
because it may produce sedation, pupil changes, euphoria, and respiratory depression, which may mask the
signs of increasing intracranial pressure.
575. When a client arrives on the labor and delivery unit, she informs the nurse that she has been having
contractions for the last 5 hours. Now the pain is constant and not cyclical as it was earlier. The nurse
considers the possibility of uterine rupture. Which of the following symptoms would be consistent with a
uterine rupture?
A. A large gush of clear fluid from the vagina
B. Systolic hypertension
C. Abdominal rigidity
D. Increased fetal movements
Answer: C
Explanation:
(A) This symptom would indicate a rupture of the membranes, which would be expected during labor.
There would be no cause for alarm if the fluid were clear.
(B) With uterine rupture and the risk of maternal shock secondary to blood loss, the most likely sign would
be hypotension indicating hypovolemic shock.
(C) In the event of a uterine rupture, an abdominal examination would likely reveal rigidity or tenderness.
(D) The most likely finding would be a decrease in fetal movement related to fetal distress due to impaired
uteroplacental blood flow. Maintaining the client on her left side would help to maximize uterine blood
flow.
576. Which of the following changes in blood pressure readings should be of greatest concern to the nurse
when assessing a prenatal client?

A. 130/88 to 144/92
B. 136/90 to 148/100
C. 150/96 to 160/104
D. 118/70 to 130/88
Answer: D
Explanation:
(A, B, C) The individual's systolic and diastolic changes are more significant than the relatively high initial
blood pressure readings.
(D) The systolic pressure went up 12 mm Hg and the diastolic pressure 18 mm Hg. This is a more
significant rise than the increases in A-C choices, and client should receive more frequent evaluations and
care.
577. A female client with major depression stated that "life is hopeless and not worth living." The nurse
should place highest priority on which of the following questions?
A. "How has your appetite been recently?"
B. "Have you thought about hurting yourself?"
C. "How is your relationship with your husband?"
D. "How has your depression affected your daily living activities?"
Answer: B
Explanation:
(A) Although eating habits are important to assess, they are less important than suicidal intent.
(B) Maintenance of the client's life is the priority; assessment of suicidal intent is imperative.
(C) Relationships and support systems are an important part of assessment, but they are less important than
suicidal intent.
(D) Daily living activities will give additional information about the level of depression, and are less
significant than suicidal intent, although this information may give additional information about the actual
plan for a suicidal attempt.
578. In counseling a client, the nurse emphasizes the danger signals during pregnancy. On the next visit,
the client identifies which of the following as a danger signal that should be reported immediately?
A. Backache
B. Leaking of clear yellow fluid from breasts
C. Constipation with hemorrhoids

D. Visual changes
Answer: D
Explanation:
(A) Backache is a common complaint during pregnancy. Proper body mechanics, pelvic rock, back rubs,
and other comfort measures should relieve the discomfort. In the presence of uterine contractions, the
backache would radiate to the lower abdomen.
(B) Colostrum is normal and can be present anytime in the second half of pregnancy.
(C) Constipation and hemorrhoids are common and do need attention, but they do not constitute a
dangerous situation.
(D) Visual changes are possibly related to PIH. The client should be assessed immediately to rule out or
prevent worsening of PIH.
579. A client has just been transferred to the floor from the labor and delivery unit following delivery of a
stillborn term infant. She is very despondent. When the nurse attempts to take her vital signs, she responds
in anger, stating, "You leave me alone. You don't care anything about me. It's people like you who let my
baby die." The nurse's best course of action is to:
A. Quietly leave her room, allowing her more private time to deal with her loss.
B. Tell her that what happened was for the best and that she is still young and can have other children.
C. Tell her how sorry you are, and let her know that her child is now a little angel in heaven.
D. Tell her how sorry you are about the loss of her baby, and acknowledge her anger as being a normal
stage of grief. Assure her that you are there to help her in any way you can.
Answer: D
Explanation:
(A) Parents do need their privacy following a loss, but the nurse still has a responsibility to provide
postpartum physical care.
(B) This is a negative statement, which is not therapeutic. The client is not concerned about future children
but is in the first stages of grief, denial, and anger.
(C) This is a negative statement, which is not therapeutic. The client does not want to hear about her baby
in heaven. She cannot believe that God could love or want her child more than she could.
(D) Acknowledging that anger is normal and beneficial will help the client to understand the normal stages
of grief. Expressing sorrow over her loss and assuring her that the support is there to take care of her
physical and emotional needs will help to promote a trusting relationship.

580. A client in active labor asks the nurse for coaching with her breathing during contractions. The client
has attended Lamaze birth preparation classes. Which of the following is the best response by the nurse?
A. "Keep breathing with your abdominal muscles as long as you can."
B. "Make sure you take a deep cleansing breath as the contractions start, focus on an object, and breathe
about 16-20 times a minute with shallow chest breaths."
C. "Find a comfortable position before you start a contraction. Once the contraction has started, take slow
breaths using your abdominal muscles."
D. "If a woman in labor listens to her body and takes rapid, deep breaths, she will be able to deal with her
contractions quite well."
Answer: B
Explanation:
(A) Lamaze childbirth preparation teaches the use of chest, not abdominal, breathing.
(B) In Lamaze preparation, every patterned breath is preceded by a cleansing breath; as labor progresses,
shallow, paced breathing is found to be effective.
(C) It is important to assume a comfortable position in labor, but the Lamazeprepared laboring woman is
taught to breathe with her chest, not abdominal, muscles.
(D) When deep chest breathing patterns are used in Lamaze preparation, they are slowly paced at a rate of
6-9 breaths/min.
581. When the nurse is evaluating lab data for a client 18-24 hours after a major thermal burn, the expected
physiological changes would include which of the following?
A. Elevated serum sodium
B. Elevated serum calcium
C. Elevated serum protein
D. Elevated haematocrit
Answer: D
Explanation:
(A) Sodium enters the edema fluid in the burned area, lowering the sodium content of the vascular fluid.
Hyponatremia may continue for days to several weeks because of sodium loss to edema, sodium shifting
into the cells, and later, diuresis.
(B) Hypocalcemia occurs because of calcium loss to edema fluid at the burned site (third space fluid).
(C) Protein loss occurs at the burn site owing to increased capillary permeability. Serum protein levels
remain low until healing occurs.

(D) Hematocrit level is elevated owing to hemoconcentration from hypovolemia. Anemia is present in the
postburn stage owing to blood loss and hemolysis, but it cannot be assessed until the client is adequately
hydrated.
582. A physician's order reads: Administer KCl 10% oral solution 1.5 mL. The KCl bottle reads 20 mEq/15
mL. What dosage should the nurse administer to the infant?
A. 1 mEq
B. 1.13 mEq
C. 2 mEq
D. Not enough information to calculate
Answer: C
Explanation:
(A) This answer is a miscalculation.
(B) This answer is a miscalculation.
(C) 1.33 mEq = 1 mL, then 1.5 mL X = 1.99, or 2 mEq.
(D) Information is adequate for calculation.
583. Prenatal clients are routinely monitored for early signs of pregnancy-induced hypertension (PIH). For
the prenatal client, which of the following blood pressure changes from baseline would be most significant
for the nurse to report as indicative of PIH?
A. 136/88 to 144/93
B. 132/78 to 124/76
C. 114/70 to 140/88
D. 140/90 to 148/98
Answer: C
Explanation:
(A) These blood pressure changes reflect only an 8 mm Hg systolic and a 5 mm Hg diastolic increase,
which is insufficient for blood pressure changes indicating PIH.
(B) These blood pressure changes reflect a decrease in systolic pressure of 8 mm Hg and diastolic pressure
of 2 mm Hg; these values are not indicative of blood pressure increases reflecting PIH.
(C) The definition of PIH is an increase in systolic blood pressure of 30 mm Hg and/or diastolic blood
pressure of 15 mm Hg. These blood pressures reflect a change of 26 mm Hg systolically and 18mm Hg
diastolically.

(D) These blood pressures reflect a change of only 8 mm Hg systolically and 8 mm Hg diastolically, which
is insufficient for blood pressure changes indicating PIH.
584. When teaching a mother of a 4-month-old with diarrhoea about the importance of preventing
dehydration, the nurse would inform the mother about the importance of feeding her child:
A. Fruit juices
B. Diluted carbonated drinks
C. Soy-based, lactose-free formula
D. Regular formulas mixed with electrolyte solutions
Answer: C
Explanation:
(A) Diluted fruit juices are not recommended for rehydration because they tend to aggravate the diarrhoea.
(B) Diluted soft drinks have a high-carbohydrate content, which aggravates the diarrhoea.
(C) Soy-based, lactose-free formula reduces stool output and duration of diarrhoea in most infants.
(D) Regular formulas contain lactose, which can increase diarrhoea.
585. In assessing cardiovascular clients with progression of aortic stenosis, the nurse should be aware that
there is typically:
A. Decreased pulmonary blood flow and cyanosis
B. Increased pressure in the pulmonary veins and pulmonary edema
C. Systemic venous engorgement
D. Increased left ventricular systolic pressures and hypertrophy
Answer: D
Explanation:
(A) These signs are seen in pulmonic stenosis or in response to pulmonary congestion and edema and
mitral stenosis.
(B) These signs are seen primarily in mitral stenosis or as a late sign in aortic stenosis after left ventricular
failure.
(C) These signs are seen primarily in right-sided heart valve dysfunction.
(D) Left ventricular hypertrophy occurs to increase muscle mass and overcome the stenosis; left
ventricular pressures increase as left ventricular volume increases owing to insufficient emptying.

586. A 17-year-old pregnant client who is gravida 1, para 0, is at 36 weeks' gestation. Based on the nurse's
knowledge of the maternal physiological changes in pregnancy, which of these findings would be of
concern?
A. Complaints of dyspnea
B. Edema of face and hands
C. Pulse of 65 bpm at 8 weeks, 73 bpm at 36 weeks
D. Hematocrit 39%
Answer: B
Explanation:
(A) Dyspnea is a common complaint during the third trimester owing to the increasing size of the uterus
and the resulting pressure against the diaphragm.
(B) Edema of the face, hands, or pitting edema after 12 hours of bed rest may be indicative of
preeclampsia and would be of great concern to the healthcare provider.
(C) An increase in heart rate of 10-15 bpm is a normal physiological change in pregnancy due to the
multiple hemodynamic changes.
(D) A hematocrit value of 39% is within the normal range. A value <35% would indicate anemia.
587. A client calls the prenatal clinic to schedule an appointment. She states she has missed three menstrual
periods and thinks she might be pregnant. During her first visit to the prenatal clinic, it is confirmed that
she is pregnant. The registered nurse (RN) learns that her last menstrual period began on June 10.
According to Nagele's rule, the estimated date of confinement is:
A. March 17
B. June 3
C. August 30
D. January 10
Answer: A
Explanation:
(A) Using Nagele's rule, count back 3 calendar months from the first day of the last menstrual period.
The answer is March 10. Then add 7 days and 1 year, which would be March 17 of the following year. (B,
C, D) This date is incorrect.

588. A male client tells his nurse that he has had an ulcer in the past and is afraid it is "flaring up again."
The nurse begins to ask him specific questions about his symptoms. The nurse knows that a symptom that
might indicate a serious complication of an ulcer is:
A. Pain in the middle of the night
B. A bowel movement every 3-5 days
C. Melena
D. Episodes of nausea and vomiting
Answer: C
Explanation:
(A) Clients with ulcers generally experience abdominal pain. It is common to have pain in the early
morning hours with an ulcer.
(B) Constipation is not a symptom associated with ulcers and would indicate a need to look at other
factors.
(C) Melena is blood in the stools. This could indicate a slow bleeding ulcer, which could result in
significant amounts of blood loss over time.(D) Nausea and vomiting may be present as a result of the
ulcer, especially if it is a gastric ulcer. This does not indicate an immediate life-threatening complication.
589. During burn therapy, morphine is primarily administered IV for pain management because this route:
A. Delays absorption to provide continuous pain relief
B. Facilitates absorption because absorption from muscles is not dependable
C. Allows for discontinuance of the medication if respiratory depression develops
D. Avoids causing additional pain from IM injections
Answer: B
Explanation:
(A) Absorption would be increased, not decreased.
(B) IM injections should not be used until the client is hemodynamically stable and has adequate tissue
perfusion. Medications will remain in the subcutaneous tissue with the fluid that is present in the
interstitial spaces in the acute phase of the thermal injury. The client will have a poor response to the
medication administered, and a "dumping" of the medication can occur when the medication and fluid are
shifted back into the intravascular spaces in the next phase of healing.
(C) IV administration of the medication would hasten respiratory compromise, if present.
(D) The desire to avoid causing the client additional pain is not a primary reason for this route of
administration.

590. The physician has prescribed metoclopramide (Reglan). When assessing the client, the nurse would
expect to find which of the following responses?
A. Increase in gastric secretions
B. Increase in peristalsis
C. Disorientation
D. Drowsiness
Answer: B
Explanation:
(A) Metoclopramide does not stimulate gastric secretions.
(B) This response is expected with metoclopramide, in addition to increasing gastric emptying.
(C) Disorientation is a symptom of metoclopramide overdose. The drug should be discontinued.
(D) Drowsiness is a symptom of metoclopramide overdose and the drug should be discontinued.
591. A female client presents to the obstetric-gynaecology clinic for a pregnancy test, the result which
turns out to be positive. Her last menstrual period began December 10, 1993. Using Nagele's rule, the
nurse estimates her date of delivery to be:
A. September 17, 1994
B. September 10, 1994
C. September 3, 1994
D. August 17, 1994
Answer: A
Explanation:
(A) According to Nagele's rule, the estimated date of delivery is calculated by adding 7 days to the date of
the first day of the normal menstrual period (December 10 + 7 days = December 17), and then by counting
back 3 months (December 17 -3 mo = September 17).
(B, C, D) These answers are incorrect.
592. The nurse writes the following nursing diagnosis for a client in acute renal failure-Impaired gas
exchange related to:
A. Decreased red blood cell production
B. Increased levels of vitamin D
C. Increased red blood cell production

D. Decreased production of renin
Answer: A
Explanation:
(A) Red blood cell production is impaired in renal failure owing to impaired erythropoietin production.
This causes a decrease in the delivery of oxygen to the tissue and impairs gas exchange.
(B) The conversion of vitamin D to its physiologically active form is impaired in renal failure.
(C) In renal failure, a decrease in red blood cell production occurs owing to an impaired production of
erythropoietin, leading to impaired gas exchange at the cellular level.
(D) The decreased production of renin in renal failure causes an increased production of aldosterone
causing sodium and water retention.
593. A 4 days postpartum client who is gravida 3, para 3, is examined by the home health nurse during her
first postpartum home visit. The nurse notes that she has a pink vaginal discharge with a serosanguineous
consistency. The nurse would most accurately chart the client's lochia as:
A. Rubra
B. Rosa
C. Serosa
D. Alba
Answer: C
Explanation:
(A) Lochia rubra is bloody with clots and occurs 1-3 days postpartum.
(B) There is no such term as lochia rosa.
(C) Lochia serosa is a pink-brown discharge with a serosanguineous consistency that occurs 4-9 days
postpartum.
(D) Lochia alba is yellow to white in color and occurs approximately 10 days postpartum.
594. A client with a head injury asks why he cannot have something for his headache. The nurse's response
is based on the understanding that analgesics could:
A. Counteract the effects of antibiotics
B. Elevate the blood pressure
C. Mask symptoms of increasing intracranial pressure
D. Stimulate the central nervous system
Answer: C

Explanation:
(A) Analgesic medication does not counteract the effects of antibiotics.
(B) Analgesic medication may lower blood pressure elevated due to anxiety.
(C) Analgesic medication, especially CNS depressants, is not given if there is danger of increasing ICP,
because neurological changes may not be apparent. Also, further depression of the CNS is contraindicated.
(D) Analgesics do not stimulate the CNS.
595. A 55-year-old client is admitted with a diagnosis of renal calculi. He presented with severe right flank
pain, nausea, and vomiting. The most important nursing action for him at this time is:
A. Intake and output measurement
B. Daily weights
C. Straining of all urine
D. Administration of O2 therapy
Answer: C
Explanation:
(A) Intake and output measurements are important but must be accompanied by straining urine.
(B) Daily weights would not provide for identification of calculi.
(C) Straining urine provides for assessment of calculi and evaluation of calculi descent through ureters and
urethra.
(D) O2 therapy should not be necessary for renal calculi.
596. A client is having a vertical partial laryngectomy, and the nurse is planning his postoperative care. A
priority postoperative nursing diagnosis for a client having a vertical partial laryngectomy would be:
A. Activity intolerance
B. Ineffective airway clearance
C. High risk for infection
D. Altered oral mucous membrane
Answer: B
Explanation:
(A) The laryngectomy client should be able to gradually increase activities without difficulty.
(B) The laryngectomy client may have copious amounts of secretions and require suctioning for the first
24-48 hours. The cannula will require cleaning even after the first 24 hours because mucus collects in it.

(C) The client does have a potential for infection, but it is not a more important nursing priority than the
ineffective airway clearance.
(D) This problem is not a more important nursing priority than ineffective airway clearance. The client's
mouth may become dry, but good oral care should take care of the dryness.
597. The therapeutic blood-level range for lithium is:
A. 0.25-1.0 mEq/L
B. 0.5-1.5 mEq/L
C. 1.0-2.0 mEq/L
D. 2.0-2.5 mEq/L
Answer: B
Explanation:
(A) This range is too low to be therapeutic.
(B) This is the therapeutic range for lithium.
(C) This range is above the therapeutic level.
(D) This range is toxic and may cause severe side effects.
598. A 19-month-old child is admitted to the hospital for surgical repair of patent ductus arteriosus. The
child is being given digoxin. Prior to administering the medication, the nurse should:
A. Not give the digoxin if the pulse is_60
B. Not give the digoxin if the pulse is_100
C. Take the apical pulse for a full minute
D. Monitor for visual disturbances, a side effect of digoxin
Answer: C
Explanation:
(A) Digoxin should not be given to adults with an apical pulse 100 bpm. With a pulse < 100 bpm, the
medication should be withheld and the physician notified.
(C) Prior to digoxin administration in both children and adults, an apical pulse should be taken for 1 full
minute. Aside from the rate per minute, the nurse should note any sudden increase or decrease in heart rate,
irregular rhythm, or regularization of a chronic irregular heart rhythm.
(D) Early indications of digoxin toxicity, such as visual disturbances, occur rarely as initial signs in
children.

599. In the coronary care unit, a client has developed multifocal premature ventricular contractions. The
nurse should anticipate the administration of:
A. Furosemide
B. Nitro-glycerine
C. Lidocaine
D. Digoxin
Answer: C
Explanation:
(A) Furosemide is a loop diuretic.
(B) Nitro-glycerine is a vasodilator.
(C) Lidocaine is the drug of choice to treat ectopic ventricular beats.
(D) Digoxin slows down the electrical impulses and increases ventricular contractions, but it does not
rapidly correct ventricular ectopy.
600. A female client is exhibiting signs of respiratory distress. Which of the following signs indicate a
possible pneumothorax?
A. Crackles or rales on the affected side
B. Bradypnea and bradycardia
C. Shortness of breath and sharp pain on the affected side
D. Increased breath sounds on the affected side
Answer: C
Explanation:
(A) With a pneumothorax, air occupies the pleural space. Crackles or rales are heard with increased fluid
or secretions and would not be present with air in the space.
(B) With a pneumothorax, the client would experience tachypnea and tachycardia to compensate for the
decrease in oxygenation.
(C) Symptoms of pneumothorax include shortness of breath, sharp pain on the affected side with
movement or coughing, asymmetrical chest expansion, and diminished or absent breath sounds on the
affected side.
(D) With a pneumothorax, breath sounds would be decreased on the affected side (indicates air in the
pleural space).

Document Details

Related Documents

person
Harper Mitchell View profile
Close

Send listing report

highlight_off

You already reported this listing

The report is private and won't be shared with the owner

rotate_right
Close
rotate_right
Close

Send Message

image
Close

My favorites

image
Close

Application Form

image
Notifications visibility rotate_right Clear all Close close
image
image
arrow_left
arrow_right